You are on page 1of 99

Incremental Analysis

MODULE 6

INCREMENTAL ANALYSIS

Basic concepts
Steps in decision making process
5. What is the first step in the decision making process?
A. Specify the criteria by which the decision is to be made.
B. Consider the strategic issues regarding the decision context.
C. Perform an analysis in which the relevant information is developed and analyzed.
D. Compare the alternatives.

7. A major accounting contribution to the managerial decision-making process in evaluating possible courses of action is to
A. assign responsibility for the decision.
B. provide relevant revenue and cost data about each course of action.
C. determine the amount of money that should be spent on a project.
D. decide which actions that the management should consider.

8. An analysis of relevant costs and relevant revenues


A. Will enable the decision maker to assess a decision‟s impact on profit
B. Is useful in assessing a variety of alternative decisions
C. Provides sufficient and complete evidence with which to make a decision
D. Answers a. and b. are correct

Pitfalls in decision making


1. When discussing the pitfalls to be avoided in decision-making, four reminders usually emerge. Which is NOT one of those reminders?
A. Ignore sunk costs.
B. Beware of allocated fixed costs; identify the avoidable costs.
C. Pay special attention to identifying and including opportunity costs.
D. Do not overlook the time value of money in short-run decisions.

1
19. Which one of the following is not a common mistake in a decision-making process?
A. Considering sunk costs as relevant.
B. Considering opportunity cost, an imputed cost, being relevant.
C. Considering fixed costs as avoidable fixed costs.
D. Unitizing fixed costs.

24. One of the behavioral problems with relevant cost analysis is the overemphasis on
short-term goals, which can lead to neglect of:
A. sales promotion C. quarterly net income results
B. expense control D. long-term strategic goals

Incremental analysis
25. Incremental analysis is the process of identifying the financial data that:
A. do not change under alternative courses of action
B. are mixed under alternative courses of action
C. change under alternative courses of action
D. no correct answer is given

48. Incremental analysis is most


useful
A. in evaluating the master budget.
B. in choosing between the net present value method and the internal rate of return method.
C. in developing relevant information for management decisions.
D. as a replacement technique for variance analysis.

Relevant information
2. Predicted future cost and revenue data that will differ among alternative courses of action are known as
A. relevant information C. marginal costs
B. direct information D. incremental costs
4. Which of the following is described as data that are pertinent to a decision?
A. qualitative characteristics C. timely information
B. accurate information D. relevant information

6. Which of the following best describes relevant information?


A. Focused on the past and differs between the alternatives under consideration.
B. Focused on the past and not related to the decision under consideration.
C. Focused on the future and differs between the alternatives under consideration.
D. Focused on the future and not related to the decision under consideration.

Application of incremental analysis


3. Incremental analysis would not be appropriate for
A. a make or buy decision.
B. an allocation of limited resource decision.
C. elimination of an unprofitable segment.
D. analysis of manufacturing variances.

Irrelevant costs
Sunk costs
9. The kind of cost that can be ignored in a short-term decision making is a(an)
A. differential cost C. sunk cost
B. incremental cost D. joint cost

30. Sunk costs are


A. Costs that increase due to a higher volume of activity or the performance of an additional activity
B. Costs that a company must incur to perform an activity at a given level, but will not be incurred if a company reduces or discontinues the
activity
C. The profits that a company forgoes by following a particular course of action
D. Costs that were incurred prior to making a decision

33. A sunk cost is:


A. a cost incurred in the past and not relevant to any future course of action.
B. an opportunity cost.
C. useful in analysis of alternative courses of action.
D. relevant to current decision making.

13. Which of the following is least likely to be a relevant item in deciding whether to replace an old machine?
A. acquisition cost of the old machine
B. outlay to be made for the new machine
C. annual savings to be enjoyed on the new machine
D. life of the new machine

Unit costs
22. Unit costs can mislead decision makers. Which of the following situations dealing with unit costs are not expected to result in a faulty analysis?
A. Unit costs used in make-or-buy decisions might include costs such as avoidable fixed costs.
B. Variable unit cost directly varies with the changes in production units.
C. Total fixed costs increase as more units are produced within the relevant range.
D. Contribution margin on products that can be manufactured in using the freed capacity is irrelevant in the decision.

Relevant costs
16. Relevant costs are
A. all fixed and variable costs
B. all costs that would be incurred within the relevant range of production
C. past costs that are expected to be different in the future
D. anticipated future costs that will differ among various alternatives

14. The Health Care Division of Piedmont Insurance employs three claims processors capable of processing 5,000 claims each. The division currently
processes 12,000 claims. The manager has recently been approached by two sister divisions. Auto Division would like the Health Care Division to
process approximately 2,000 claims. Property Division would like the Health Care Division to process approximately 5,000 claims. The Health Care
Division would be compensated by Auto Division or Property Division for processing these claims. Assume that these are mutually exclusive
alternatives. Claims processor salary cost is relevant for
A. Auto Division alternative only
B. Property Division alternative only
C. both Auto Division and Property Division alternatives
D. neither Auto Division nor Property Division alternatives

Differential costs
31. The difference in cost between or among various alternative courses of action appropriately describes a(an):
A. differential cost C. constraint
B. ad hoc discount D. scarce resource

Opportunity cost
10. An important concept in decision making is described as “the contribution to income that is forgone by not using a limited resource in its best alternative
use.” This concept is called
A. Marginal cost C. Incremental cost
B. Cost outlay D. Opportunity cost

11. An “opportunity cost” is


A. the difference in total costs that results from selecting one alternative instead of another
B. the profit forgone by selecting one alternative instead of another
C. a cost that may be saved by not adopting an alternative
D. a cost that may be shifted to the future with little or no effect on current operations

12. The best characterization of an opportunity cost is that it is


A. relevant to decision making but is not usually reflected in accounting records
B. not relevant to decision making and is not usually reflected in accounting records
C. relevant to decision making and is usually reflected in accounting records
D. not relevant to decision making and is usually reflected in accounting records

18. The potential benefit that may be obtained from following an alternative course of action is called
A. opportunity benefit C. relevant cost
B. opportunity cost D. sunk cost

26. Opportunity cost is the


A. cash outlay required to implement an alternative.
B. difference in total costs between the alternatives.
C. maximum available contribution to profit that is given up when using limited resources for another purpose.
D. fixed cost avoided when a product, department, or business unit is abandoned.

28. Opportunity costs are


A. Costs that increase due to a higher volume of activity or the performance of an additional activity
B. Costs that a company must incur to perform an activity at a given level, but will not be incurred if a company reduces or discontinues the
activity
C. The profits that a company forgoes by following a particular course of action
D. Costs that were incurred prior to making a decision

27. Using opportunity cost to analyze the income effects of a given alternative is referred to as
A. engineering analysis C. account analysis
B. mixed-cost analysis D. differential analysis

Avoidable
15. A fixed cost is relevant if it is
A. future cost C. avoidable
B. sunk D. a product cost

17. Which of the following is (are) a true statement(s) about cost behaviors in incremental analysis?
I. Fixed costs will not change between alternatives.
II. Fixed costs may change between alternatives.
III. Variable costs will always change between alternatives.
A. I C. III
B. II D. II and III
29. Avoidable costs are
A. Costs that increase due to a higher volume of activity or the performance of an additional activity
B. Costs that a company must incur to perform an activity at a given level, but will not be incurred if a company reduces or discontinues the
activity
C. The profits that a company forgoes by following a particular course of action
D. Costs that were incurred prior to making a decision

Out-of-pocket costs
23. Which of the following is a cost that requires a future outlay of cash that is relevant for future decision-making?
A. Opportunity cost C. Out-of-pocket cost
B. Relevant benefits D. Incremental revenue

Sensitivity analysis
20. Sensitivity analysis is useful in decision making when:
A. there is a degree of uncertainty about the relevant data.
B. there is an opportunity cost included in the analysis.
C. sunk cost is included in the analysis.
D. the analysis is subject to a review by the management.

21. To determine the possible outcome in a decision analysis if a key prediction or assumption proves to be wrong, managers will use:
A. sensitivity analysis. C. incremental analysis.
B. total analysis. D. regression analysis.

Make-or-buy decision
Qualitative Considerations
38. Which of the following elements of the value chain should be considered when deciding whether to make or buy a component needed for
production?
A. Marketing C. Manufacturing
B. Distribution D. all of these choices

Make decision
34. Manufacturing parts internally by a company causes:
A. the company to be dependent upon suppliers for timely delivery of parts
B. the quality of the parts to be under the control of the company
C. lower parts costs to be assured
D. a company's operations to be more efficient than when the parts are purchased from suppliers

44. A company should decide to make, rather than buy, a part required for their product, if
A. The company‟s production facility is at full capacity
B. The relevant cost per-unit of making the part exceeds the per-unit relevant costs of purchasing the part
C. The supplier of the part can produce a higher-quality part
D. The supplier of the part has questionable reliability

Buy decision
35. In any make or buy decision confronting a company, which of the following factors should be considered?
A. Can the supplier provide a sufficient quantity to meet the company's current and future needs?
B. Do the supplier's items meet product and quality specifications?
C. Is the supplier reliable?
D. All of the above should be considered.

41. Which of the following qualitative factors favors the buy choice in a make or buy decision for a part?
A. maintaining a long-term relationship with suppliers
B. quality control is critical
C. utilization of idle capacity
D. part is critical to product

Relevant costs
Fixed costs
36. Within the context of the make or buy decision, when are fixed costs relevant?
A. Fixed costs are always relevant
B. Fixed costs are never relevant
C. Fixed costs are relevant when they differ among alternatives
D. It cannot be determined without closely examining each particular situation

37. In a make or buy decision:


A. Only variable costs are relevant.
B. Fixed costs that can be avoided in the future are relevant.
C. Fixed costs that will continue regardless of the decision are relevant.
D. Only conversion costs are relevant.

Opportunity costs
39. In a make-or-buy decision, which of the following is true?
A. Variable costs are the only relevant costs.
B. Allocated fixed costs are relevant.
C. Alternative uses of space and machinery are relevant.
D. Making the product is the correct decision when there is idle capacity.

40. The opportunity cost of making a component part in a factory with excess capacity for which there is no alternative use is
A. the total manufacturing cost of the component.
B. the total variable cost of the component.
C. the fixed manufacturing cost of the component.
D. zero.

46. The cost of not receiving rent from a space because you decide to make the part rather than buying it from an outside supplier is considered
a(an)
A. sunk cost C. opportunity cost
B. future cost D. fixed cost

47. In a make-or-buy decision, an opportunity cost that should be considered is the:


A. income that could be generated from idle production space.
B. total costs to produce the item
C. variable costs to produce the item
D. fixed costs to produce the item
Decision rule
42. Haribon Company is faced with a make-or-buy decision. Haribon should agree to buy the part from a supplier provided the price is less than
Haribon‟s
A. total costs
B. variable production costs plus avoidable fixed production costs
C. total manufacturing costs
D. variable costs

84. A company owns equipment that is used to manufacture important parts for its production process. The company plans to sell the equipment for
P10,000 and to select one of the following alternatives:
(1) acquire new equipment for P80,000
(2) purchase the important parts from an outside company at P4 per part.
The company should quantitatively analyze the alternatives by comparing the cost of manufacture the parts
A. Plus P80,000 to the cost of buying the parts less P10,000.
B. to the cost of buying the parts less P10,000.
C. Less P10,000 to the cost of buying the parts.
D. To the cost of buying the parts.

Special order decision


Process
49. In making a special order decision, management should:
A. compute a reasonable sales price for items not normally produced.
B. consider additional overhead cost.
C. consider normal and relevant costs.
D. All of the above.

52. Which of the following factors should be considered in deciding whether to accept a special order?
A. the sales price of the product or service
B. the production capacity of the company
C. the impact on regular customers
D. all of these choices

Irrelevant cost
83. In considering a special order that will enable a company to make a use of presently idle capacity, which of the following costs would be
irrelevant.
A. Materials C. Direct labor
B. Depreciation D. Variable OH

54. Given the following list of costs, which one should be ignored in a decision to produce additional units of product for a factory that is operating
at less than 100% capacity, and the additional business will not use up the remainder of the plant capacity?
A. Direct material cost per unit C. Fixed selling expenses
B. Direct labor cost per hour D. Variable selling expenses

Relevant costs
Long-run decision
58. The sales price of a product, in the long run, must be enough to cover what type of costs?
A. Designing costs C. Servicing costs
B. Marketing costs D. All of the above

Opportunity costss
50. An opportunity cost commonly associated with a special order is
A. the contribution margin on lost sales
B. the variable costs of the order
C. additional fixed cost that is related to the increased output
D. any of the above

53. Operating at or near full capacity will require a firm considering a special order to recognize the:
A. opportunity cost arising from lost sales
B. value of full employment
C. time value of money
D. need for good management
Decision rule
82. Production of a special order will increase gross profit when the additional revenue from the special order is greater than
A. The nonvariable costs incurred in producing the order.
B. The direct material and labor costs in producing the order.
C. The fixed costs incurred in producing the order.
D. The marginal cost of producing the order.

51. If the firm is operating under capacity, the minimum special order price should be high enough to cover:
A. all variable costs and incremental fixed costs associated with the special order minus foregone contribution margin on regular units not
produced.
B. variable and incremental fixed costs associated with the special order and a profit margin.
C. limited variable costs associated with the special order.
D. neither variable nor fixed costs associated with the special order.

57. Green Giant Foods has some excess manufacturing capacity that it can leave idle, use to produce its own boxes for frozen foods, or use to process
another company‟s frozen foods. It will be more profitable for Green Giant to process the competitor‟s frozen foods as long as the net cost is
A. greater than both the cost to buy the boxes and the cost to leave the plant idle.
B. less than the cost to leave the plant idle and greater than the cost to buy the boxes.
C. greater than the cost to leave the plant idle and lower than the cost to buy boxes from a supplier.
D. less than both the cost to leave the plant idle and the cost to make or buy the boxes.

Minimum acceptable price


With excess capacity
55. If there is excess capacity, the minimum acceptable price for a special order must cover
A. variable costs associated with the special order.
B. variable and fixed manufacturing costs associated with the special order.
C. variable and incremental fixed costs associated with the special order.
D. variable costs and incremental fixed costs associated with the special order plus the contribution margin usually earned on regular units.

At full capacity
56. If a firm is at full capacity, the minimum special order price must cover
A. variable costs associated with the special order.
B. variable and fixed manufacturing costs associated with the special order.
C. variable and incremental fixed costs associated with the special order.
D. variable costs and incremental fixed costs associated with the special order plus foregone contribution margin on regular units not produced.

Product life cycle


45. A product life cycle includes the phases of
A. research and development and design C. marketing and distribution
B. purchasing and production D. all of the above

Product pricing
Variable cost approach
60. Managers who often make special pricing decisions are more likely to use which of the following cost concepts in their work?
A. Total cost. C. Variable cost.
B. Product cost. D. Fixed cost.

Cost-plus approach
59. In using the variable cost concept of applying the cost-plus approach to product pricing, what is included in the markup?
A. Total costs plus desired profit.
B. Desired profit.
C. Total selling and administrative expenses plus desired profit.
D. Total fixed manufacturing costs, total fixed selling and administrative expenses, and desired profit.

62. Which of the following is NOT a cost concept commonly used in applying the cost-plus approach to product pricing?
A. Total cost concept. C. Variable cost concept.
B. Product cost concept. D. Fixed cost concept.

63. The cost-plus pricing formula that takes into consideration all costs -- fixed, variable, and manufacturing, as well as selling and administrative costs -- is
called the percentage of
A. full costs. C. total variable costs.
B. variable manufacturing costs. D. absorption costs.

Target pricing
43. The concept of target pricing is employed when:
A. a company wishes to set price in order to capture a predetermined market share.
B. a price is pre-set by market conditions.
C. a company wishes to meet marketing goals.
D. All of the above.

Target cost approach


61. In contrast to the total product and variable cost concepts used in setting seller's prices, the target cost approach assumes that:
A. a markup is added to total cost. C. a markup is added to variable cost.
B. selling price is set by the marketplace. D. a markup is added to product cost.

Sell-as-is-or-process further
Joint products
67. Two or more manufactured products that have significant sales values and are not uniquely identifiable as individual products until the split-off point are
called
A. common products. C. co-mingled products.
B. joint products. D. cooperative products.

Relevant costs
Incremental revenue
32. Incremental revenue is:
A. a difference in costs between two decisions.
B. a concession based on competitive influences.
C. additional revenue across decision choices from potential sales.
D. the difference between selling price and variable costs.

Cost to process further


65. Which of the following costs is relevant in deciding whether to sell joint products at split-off or process them further?
A. The unavoidable costs of further processing.
B. The additional costs of further processing.
C. The variable costs of operating the joint process.
D. The cost of materials used to make the joint products.

68. What are the manufacturing costs incurred beyond the split-off point called?
A. Separable costs. C. Severance costs.
B. Joint costs. D. Common costs.

Decision rule
64. How does a company determine whether to sell a product “as is” or process it further?
A. If the costs to process further exceed the costs of current production, the product should be sold „as is.”
B. If the costs to process further exceed the costs of current production, the product should be processed further.
C. If the increase in revenue from selling the product after further processing is greater than the additional costs incurred in further processing, the
company should opt for further processing.
D. If the revenues generated by processing the product further exceed the revenues from selling the product “as is,” the company should process
further.

Keep-or-drop decision
Strategic considerations
66. The decision to keep or drop products or services involves strategic consideration of the:
A. potential impact on remaining products or services
B. impact on employee morale
C. growth potential of the firm
D. All of the above answers are correct

Goal
78. The goal in deciding whether to add or drop products, services, or departments is to obtain the greatest
A. reduction in total costs.
B. contribution possible to cover unavoidable costs.
C. increase in sales revenues.
D. decrease in direct fixed costs.

Irrelevant cost
80. Which of the following should not enter into decision of whether to drop product?
A. Unavoidable costs
B. Avoidable costs
C. Revenue that would be lost
D. Nonfinancial impacts of the decision

Decision rule
79. As long as its marginal cost is lower than its marginal revenue, a company should
A. suspend additional production and sales activities.
B. perform a cost-benefit balance analysis before producing and selling additional products.
C. engage in additional production and sales activities.
D. examine cost behaviors and develop a cost function to measure the cost of future production.

Short-run profit maximization


Factors affecting sales mix
70. Which of the following is an important factor affecting the sales mix of any company?
A. organizational advertising expenditures
B. organizational sales force compensation plan
C. product selling price
D. All of the above

To relax a constraint
73. Which of the following will relax a constraint?
A. Outsourcing all or part of the bottleneck operation
B. Working overtime at the bottleneck operation
C. Retraining employees and shifting them from the bottleneck
D. A and B, only

Decision rule
76. A product mix decision involves
A. Influencing the sales volume mix of the products to minimize cost.
B. Influencing the sales volume mix of the products to maximize revenue.
C. Producing the maximum amount of items that provide the highest contribution margin.
D. Producing the maximum amount of items that carry the lowest per-unit cost.

71. A useful device for solving production problems involving multiple products and limited resources is:
A. gross sales per unit of product C. net profit per unit of product
B. contribution per unit of scarce resource D. total benefit

72. When there is only one production constraint and excess demand, it is generally best to focus production and sales on the product with the
highest:
A. Contribution per unit of scarce resource C. Contribution margin in pesos
B. Margin of Safety D. Operating Leverage

69. When there is one scarce resource, the product that should be produced first is the product with the highest
A. contribution margin per unit of the scarce resource.
B. sales price per unit of scarce resource.
C. demand.
D. contribution margin per unit.

74. Uranus Company has 2 products that use the same manufacturing facilities and cannot be subcontracted. Each product has sufficient orders to utilize
the entire manufacturing capacity. For short-run profit maximization, Uranus should manufacture the product with the
A. Lower total manufacturing costs for the manufacturing capacity.
B. Lower total variable manufacturing costs for the manufacturing capacity.
C. Greater gross profit per hour of manufacturing capacity.
D. Greater contribution margin per hour of manufacturing capacity.
75. Profit can be maximized by producing products with the highest
A. selling price
B. contribution margin
C. contribution margin per unit of items that are best sellers
D. contribution per unit of the constraining resource

77. A company should advertise those products that


A. Require the lowest commitment of resources to produce
B. Have the largest total contribution margin
C. Can be outsourced
D. Have the largest total contribution margin after deducting the cost of the ad campaign

Pitfall
81. The major pitfall in the contribution margin approach to pricing is
A. its failure to recognize fixed costs.
B. its failure to recognize depreciation expense.
C. its inability to control waste.
D. its inability to recognize financing costs of the production in question.

PROBLEMS:
Incremental (decremental) cost
i. For the year ended April 30, 2007, Salmo Company incurred direct costs of P800,000 based on a particular course of action. Had a different course of
action been taken, direct costs would have been P650,000. In addition, Salmo‟s fixed costs during the fiscal year were P110,000.
The incremental (decremental) cost was:
A. P 40,000 C. P 150,000
B. P( 40,000) D. P(150,000)

Opportunity cost
ii. Luzon Fabricators, Inc. estimates that 60,000 special components will be used in the manufacture of a specialty steel window for the whole next
year. Its supplier quoted a price of P60 per component. Luzon prefers to purchase 5,000 units per month, but its supplier could not guarantee this
delivery schedule. In
order to ensure availability of these components, Luzon is considering the purchase of all the 60,000 units at the beginning of the year. Assuming Luzon
can invest cash at 8%, the company‟s opportunity cost of purchasing all the 60,000 units at the beginning of the year is
A. P132,000 C. P144,000
B. P150,000 D. P264,000

Defective/obsolete inventory
Incremental net income
iii. Sieney & Company has 24,000 defective units of a product that cost P8 per unit to manufacture, and can be sold for P4 per unit. These units can be
reworked for P2 per unit and sold at their full price of P12 each. If Sieney reworks the defective units, how much incremental net income will
result?
A. P144,000 C. P 72,000
B. P 96,000 D. P 48,000

Minimum price
iv. Joji Company manufactures and sell FM radios. Information on last year‟s operations (sales and production of the 2006 model)
follows: Selling price P300
Cost per unit:
Direct materials 70
Direct labor 40
Overhead (50% variable) 60
Selling costs (40% variable) 100
Production in units 10,000
Sales in units 9,500
At this time (May 2007), the 2007 model is in production and it renders the 2006 model radio obsolete. A foreign firm is willing to purchase the
obsolete products at a net price of P140 each. If the remaining 500 units of the 2006 model radios are to be sold through regular channels, what
is the minimum price the company would accept for the radios?
A. P300 C. P270
B. P180 D. P 40

Special order
Unit relevant cost
v. Venus Company, a manufacturer of lamps, budgeted sales of 400,000 lamps at P20 per unit for the year. Variable manufacturing costs were budgeted at
P8 per unit, and fixed manufacturing costs at P 5 per unit. A special order offering to buy 40,000 lamps for P11.50 each was received by Venus in
April. Venus has sufficient plant capacity to manufacture the additional quantity of lamps; however, the production would have to be done by the present
work force on an overtime-basis at an estimated additional cost of P1.50 per lamp. Venus will not incur any selling expenses as a result of the special
order. Venus Company would have a unit relevant cost of
A. P 8.00 C. P 9.50
B. P13.00 D. P14.50

vi. Wawa Enterprises has the capacity to produce 10,000 bearings, but operates at 90% of capacity. Bearings normally sell for P60 each, and cost an
average of P50 to make, including a share of the monthly fixed costs of P180,000. Ilog Corp has offered to buy 1,000 bearings at P40 each. What is the
relevant cost per unit?
A. P 20 C. P 40
B. P 30 D. P 50

Total relevant cost


vii.Intellectual Co. recently received an order for a product that it does not normally produce. Since the company has excess production capacity,
management is considering accepting the order. In analyzing the decision, the assistant controller is compiling the relevant costs of producing the
order.
The special order requires 1,000 kilograms of powdered Nitrocide, a solid chemical regularly used in the company‟s products. The current stock of
Nitrocide is 8,000 kilograms at a book value of P8.10 per kilogram. If the special order is accepted, the firm will be forced to restock powdered
Nitrocide earlier than expected, at a predicted cost of P8.70 per kilogram. Without the special order, the purchasing manager predicts that the
price will be P8.30, when normal restocking takes place. Any order of the Nitrocide must be in 5,000 kilograms.
What is the relevant cost of powdered Nitrocide to be included in the special
order? A. P 8,700 C. P10,300
B. P 8,300 D. P43,500

Incremental cost
viii. Balagtas & Company expects to incur the following costs at the planned production level of
10,000 units: Direct materials P100,000
Direct labor 120,000
Variable overhead 60,000
Fixed overhead 30,000
The selling price is P50 per unit. The company currently operates at full capacity of 10,000 units. Capacity can be increased to 13,000 units by
operating overtime. Variable costs increase by P14 per unit for overtime production. Fixed overhead costs remain unchanged when overtime
operations occur. Balagtas has received a special order from Florante, Inc. who has offered to buy 2,000 units at P45 each.
What is the incremental cost associated with this special
order? A. P42,000 C. P31,000
B. P84,000 D. P62,000

Minimum acceptable price


ix. Brace Co. has considerable excess manufacturing capacity. A special job order‟s cost sheet includes the following applied manufacturing overhead
costs: Variable costs P56,250
Fixed costs 45,000
The fixed costs include a normal P6,800 allocation for in-house design costs, although no in-house design will be done. Instead, the special job will require
the use of external designers costing P13,750.
What is the minimum acceptable price for the
job? A. P 63,050 C. P101,250
B. P 70,000 D. P108,200

x. The cost to produce 24,000 units at 70% capacity consists of:


Direct materials P360,000
Direct labor 540,000
Factory overhead, all fixed 290,000
Selling expense (35% variable, 65% fixed) 240,000
What unit price would the company have to charge to make P22,500 on a sale of 1,500 additional units that would be shipped out of the normal
market area?
A. P 51 C. P 41
B. P 56 D. P 50

xi. Kaila Company‟s unit cost of manufacturing and selling a given item at an activity level of 10,000 units per month
are: Manufacturing costs
Direct materials P39
Direct labor 6
Variable overhead 8
Fixed overhead 9
Selling expenses
Variable 30
Fixed 11
The company desires to seek an order for 5,000 units from a foreign customer. The variable selling expenses will be reduced by 40%, but the
fixed costs for obtaining the order will be P20,000. Domestic sales will not be affected by the order.
The minimum break-even price per unit to be considered on this special sale is
A. P 71 C. P 69
B. P 75 D. P 84

xii.Chrisy Company sells a product for P18 per unit and the standard cost card for the product shows the following
costs: Direct materials P 1.00
Direct labor 2.00
Overhead (80% fixed) 7.00
Total P10.00
Chrisy received a special order for 1,000 units of the product. The only additional cost to Chrisy would be foreign import taxes of P1 per unit. If
Chrisy is able to sell all of the current production domestically, what would be the minimum sales price that Chrisy would consider for this special
order?
A. P 18 C. P 17
B. P 19 D. P 11

xiii. De Silva Co. is a manufacturer of industrial components. One of their products that is used as a subcomponent in auto manufacturing is KB69. This
product has the following financial structure per unit:
Selling price P150
Direct materials P 20
Direct labor 15
Variable manufacturing overhead 12
Fixed manufacturing overhead 30
Variable shipping and handling 3
Fixed selling and administrative 10
Total P
90
De Silva is operating at full capacity. It has received a special, one-time, order for 1,000 KB69 parts. The next best alternative use of the excess capacity
is to produce LB46, resulting in a contribution margin of P10,000. The minimum price that is acceptable for this one-time special order is
A. P 60 C. P 70
B. P 87 D. P100

xiv. Sylvania Company. is currently operating at a loss of P15,000. The sales manager has received a special order for 5,000 units of product, which
normally sells for P35 per unit. Costs associated with the product are: direct material, P6; direct labor, P10; variable overhead, P3; applied fixed overhead,
P4; and variable selling expenses, P2. The special order would allow the use of a slightly lower grade of direct material, thereby lowering the price per unit
by P1.50 and selling expenses would be decreased by P1. If Sylvania wants this special order to increase the total net income for the firm to
P25,000, what sales price must be quoted for each of the 5,000 units?
A. P18.50 C. P29.00
B. P24.50 D. P26.50

Maximum lost regular sales


xv.Chua Company sells a product for P20 with variable cost of P8 per unit. Chua could accept a special order for 1,000 units at P14. If Chua
accepted the order, how many units could it lose at the regular price before the decision become unwise?
A. 1,000 units C. 500 units
B. 200 units D. 0 units

xvi. Filamer Company currently sells 1,000 units of product M for P2 each. Variable costs are P1.50. A discount store has offered P1.70 per unit for
400 units of product M. The managers believe that if they accept the special order, they will lose some sales at the regular price. Determine the number
of units they could lose before the order become unprofitable.
A. 200 units. C. 400 units.
B. 160 units. D. 500 units

Effect on profit of accepting the order


xvii. You have been approached by a foreign customer who wants to place an order for 15,000 units of Product C at P22.50 a unit. You currently
sell this item for P39 a unit, and the item has a cost of P29 a unit. Further analysis reveals that you will not be paying sales commission of P2.50
a unit on this sales and its
packaging requirement will save you an additional P1.50 per unit. However, the additional graphics required on this job will cost you P30,000. Note also
that fixed costs amounting to P400,000 for the production of 50,000 of such products by the firm will not change. You decide to accept this order,
but another customer who buys an average of 2,000 units for the period wants to pay you P22.50 rather than the regular price of P39 a unit.
Profit will
A. increase profit by P19,500 C. increase profit by P52,500
B. increase profit by P16,500 D. decrease profit by P52,500

xviii. The Thermo Company has received a special order for 300 units of product X for P6 a unit. It usually sells for P9.50 a unit with a cost of P7.50
a unit inclusive of 75 cents a unit as sales commission that will not be paid on this order. The cost also includes P3 in manufacturing overhead, was two-
third of which is for the fair share of depreciation, rent, utilities and supervisor's salary. The latter‟s (supervisor's salary) accounts for one-half of this
amount. Assuming that excess capacity is available, and this order requires a mold that costs P150, accepting the order will increase
A. loss by P225 C. gain by P225
B. loss by P375 D. gain by P375

xix. Alejar Company manufactures a product with a unit variable cost of P50 and a unit sales price of P88. Fixed manufacturing costs were P240,000 when
10,000 units were produced and sold. The company has a one-time opportunity to sell an additional 3,000 units at P70 each in a foreign market.
This special sale would not affect its present sales. If the company has sufficient capacity to produce the additional units, acceptance of the special order
would affect net income as follows:
A. Income would decrease by P 12,000.
B. Income would increase by P 12,000.
C. Income would increase by P210,000.
D. Income would increase by P 60,000.

xx.KC Industries manufactures a product with the following costs per unit at the expected production of 30,000
units. Direct materials P 4
Direct labor 12
Variable manufacturing overhead 6
Fixed manufacturing overhead 8
The company has the capacity to produce 40,000 units. The product regularly sells for P40. A wholesaler has offered to pay P32 a unit for 2,000
units. If the firm accepts the special order the effect on its operating income would be a
A. P20,000 increase C. P4,000 increase
B. P16,000 decrease D. P 0 effect

xxi. Louderhead Company makes bull-repellent scent according to a traditional Western recipe, which normally sells at P90 per unit. Normal production
volume is 10,000 ounces per month. Average cost is P50 per ounce, of which P20 is direct material and P10 is variable conversion cost. This product is
seasonal. After July, demand for this product drops to 6,000 ounces monthly. In November, Garrison Co. offers to buy 1,500 ounces for P60,000. If
Louderhead accepts the order, it must design a special label for Garrison at a cost of P5,000. Each label will cost P2.50 to make and apply.
Louderhead should:
A. accept the order, at a gain of P6,250
B. reject the order, at a loss of P18,750
C. reject the order, at a loss of P23,750
D. accept the order, at a gain of P11,250

Question Nos. 68 and 69 are based on the following information:


The Disk Division of Systems Specialist Company produces a high quality computer disks. Unit production costs (based on capacity production of 100,000
units per year) follow:
Direct materials P50
Direct labor 20
Overhead (20% variable) 10
Other information:
Sales price 100
SG & A costs (40% variable) 15
The Disk Division is operating at a level of 70,000 chips per year.

xxii. What is the minimum price that the division would consider on a “special order” of 1,000 disks to be distributed through normal channels?
A. P 72 C. P 81
B. P 78 D. P 6

xxiii. Assuming that that the Disk Division is producing and selling at capacity. What is the minimum selling price that the division would consider on a
“special order” of 1,000 chips on which no variable period costs would be incurred?
A. P100 C. P 94
B. P 72 D. P 90
Make-or-buy decision
Relevant costs
xxiv. For the past 12 years, the JLO Company has produced the small electric motors that fit into its main product line of dental drilling equipment. As
materials costs have steadily increased, the controller of the JLO Company is reviewing the decision to continue to make the small motors and has
identified the following facts:
1) The equipment which is used to manufacture the electric motors has a book value of P1,500,000.
2) The space being occupied now by the electric motor manufacturing department could be used to eliminate the need for storage space which is
presently being rented.
3) Comparable units can be purchased from an outside supplier for P597.50.
4) Four of the people who work in the electric motor manufacturing department would be terminated and given eight weeks of separation pay.
5) A P750,000 unsecured note is still outstanding on the equipment that is being used in the manufacturing
process. Which of the items above are relevant to the decision that the controller has to make?
A. 1, 2, 4, and 5 C. 1, 3, 4, and 5
B. 1, 3, and 4 D. 2, 3, and 4

Relevant cost to make


xxv. ELM Electronics has the following standard costs and other data:
Part Beta Part Zeta
Direct materials P 4.00 P80.00
Direct labor 10.00 47.00
Factory overhead 40.00 20.00
Unit standard cost P54.00 P147.00
Units needed per 6,000 8,000
year
Machine hours per 4 2
unit
Unit cost if P50.00 P150.00
purchased
In the past years, ELM has manufactured all of its required components; however, this year only 30,000 hours of otherwise idle machine time can be
devoted to
the production of components. Accordingly, some of the parts must be purchased from outside suppliers. In producing the parts, factory overhead is
applied at P10 per standard machine hour. Fixed capacity costs that will not be affected by any make-or-buy decision represent 60% of the applied
overhead.
The available 30,000 machine hours are to be scheduled so that ELM realizes maximum potential cost savings. The relevant unit production costs that
should be considered in the decision to schedule machine time are:
A. P54.00 for Beta and P147.00 for Zeta C. P14.00 for Beta and P127.00 for Zeta
B. P50.00 for Beta and P150.00 for Zeta D. P30.00 for Beta and P135.00 for Zeta

Maximum buy price


xxvi. The following are a company‟s monthly unit costs to manufacture and market a particular
product. Manufacturing Costs:
Direct materials P2.00
Direct labor 2.40
Variable indirect 1.60
Fixed indirect 1.00

Marketing Costs:
Variable 2.50
Fixed 1.50
The company must decide to continue making the product or buy it from an outside supplier. The supplier has offered to make the product at a level of
quality that the company prescribes. Fixed marketing costs would be unaffected, but variable marketing costs would continue at 30% if the company were
to accept the proposal.
What is the maximum amount per unit that the company can pay the supplier without decreasing its operating
income? A. P8.50 C. P7.75
B. P6.75 D. P5.25

xxvii. Sinta Company can make 1,000 units of a necessary component with the following
costs: Direct Materials P64,000
Direct Labor 16,000
Variable Overhead 8,000
Fixed Overhead ?
The company can purchase the 1,000 units externally for P104,000. An analysis shows that at this external price, the company is indifferent between
making or buying the part. Sinta Company could avoid P6,000 in fixed overhead costs if it acquires the components externally. If cost
minimization is the major consideration and the company would prefer to buy the components, what is the maximum external price that Sinta
Company would accept to acquire the 1,000 units externally?
A. P102,000. C. P 96,000.
B. P 94,000. D. P 88,000.

xxviii. Almeda's Shop can make 1,000 units of a necessary component with the following
costs: Direct Materials P64,000
Direct Labor 16,000
Variable Overhead 8,000
Fixed Overhead ?
The company can purchase the 1,000 units externally for P104,000. None of Almeda Company's fixed overhead costs can be reduced, but another
product could be made that would increase profit contribution by P16,000 if the components were acquired externally. If cost minimization is the major
consideration and the company would prefer to buy the components, what is the maximum external price that Almeda Company would be willing
to accept to acquire the 1,000 units externally?
A. P 86,000. C. P 96,000.
B. P110,000. D. P104,000.

Effect of make decision


xxix. A business is operating at 90% of capacity and is currently purchasing a part which is being used in its manufacturing operations for P15 per unit. The
unit cost for the business to make the part is P20, including fixed costs, and P12, not including fixed costs. If 30,000 units of the part are normally
purchased during the year but could be manufactured using unused capacity, what would be the amount of differential cost, increase or decrease, from
making the part rather than purchasing it?
A. P150,000 cost increase C. P150,000 cost decrease
B. P 90,000 cost decrease D. P 90,000 cost increase

xxx. Alfaro's Manufacturing Company can make 100 units of a necessary component part with the following
costs: Direct Materials P80,000
Direct Labor 13,000
Variable Overhead 40,000
Fixed Overhead 27,000
If Alfaro's Manufacturing Company can purchase the component externally for P145,000 and only P4,000 of the fixed costs can be avoided, what is the
correct “make or buy” decision?
A. Make and save P8,000 C. Make and save P20,000
B. Buy and save P8,000 D. Buy and save P20,000

Effect of buy decision


On fixed overhead cost
xxxi. Sisa's Shop can make 1,000 units of a necessary component with the following
costs: Direct Materials P64,000
Direct Labor 16,000
Variable Overhead 8,000
Fixed Overhead ?
The company can purchase the 1,000 units externally for P104,000. The unavoidable fixed costs are P5,000 if the units are purchased externally. An
analysis shows that at this external price, the company is indifferent between making or buying the part. What are the fixed overhead costs of making the
component?
A. P21,000. C. P11,000.
B. P16,000. D. Cannot be determined.

On income
xxxii. Sylvan Processing Company is considering whether to make 2,000 units of product Whirl which costs P16 a unit or buy it from outside for P15
a unit. A further analysis shows that if product Whirl is outsourced, fixed costs of P8,000 attributable to this product will be reduced by 25%.
If the product is outsourced, Sylvan will
A. Decrease profit by P2,000 C. Increase profit by P2,000
B. Decrease profit by P4,000 D. Increase profit by P4,000

xxxiii. Sylvan Processing Company is considering whether to make 2,000 units of product Whirl which costs P16 a unit or buy it from outside for P15
a unit. A further analysis shows that if product Whirl is outsourced, fixed costs of P8,000 attributable to this product will be reduced by 25%. If
Sylvan Processing Company purchased the product Whirl, the space could be rented out for P6,000. If the product is outsourced, profit would
A. decrease, P2,000 C. increase, P2,000
B. decrease, P4,000 D. increase, P4,000

xxxiv.It costs P450,000 to make 15,000 units of a part in this plant. This cost includes material of P90,000, direct labor of P120,000, variable
overhead of P15,000, and P225,000 in fixed overhead inclusive of P45,000 in depreciation and common overhead allocation of P150,000. The balance is
for the section supervisor's salary. The part can be purchased for P20 a unit. If the part is purchased, the space released can be rented for P65,000. If the
part is purchased, the company will
A. lose P20,000 C. gain P20,000
B. lose P45,000 D. gain P45,000

xxxv. Lane Co. manufactures ballpoint pens. Another manufacturer has offered to supply Lane with the 5,000 ink cartridges that it needs annually. The cost
to buy the cartridges would be P15 each. In producing its own cartridges, Lane has incurred P10 in fixed costs and P8 in variable costs. If Lane
buys the cartridges, its net income will:
A. not change C. increase by P35,000
B. decrease by P35,000 D. increase by P25,000

xxxvi.The Rainbow Company manufactures Part No. 498 for use in its production cycle. The cost per unit if 20,000 units of Part No. 498 are manufactured
are as follows:
Direct materials P6
Direct labor 30
Variable overhead 12
Fixed overhead applied 16
Total unit cost P64
The Reeves Company has offered to sell 20,000 units of part No. 498 to Rainbow for P60 per unit. Rainbow will make the decision to buy the part from
Reeves if there is a savings of P25,000 for Rainbow. If Rainbow accepts Reeves‟s offer, P9 per unit of the fixed overhead applied would be totally
eliminated. Furthermore, Rainbow has determined that the released facilities could be used to save relevant costs in the manufacture of part No. 575. In
order to have a savings of P25,000, the amount of the relevant costs that would be saved by using the released facilities in the manufacture of Part No.
575 would have to be A. P 80,000 C. P125,000
B. P 85,000 D. P140,000

xxxvii. Leis Manufacturing Co. uses 10 units of Part Number WS73 each month in the production of computer printer. The unit cost to
manufacture one unit of
WS73 is presented below.
Direct materials P 1,000
Materials handling (20% of direct material cost) 200
Direct labor 8,000
Manufacturing overhead (150% of direct labor) 12,000
Total manufacturing cost P21,200
Material handling represents the direct variable costs of the Receiving Department that are applied to direct materials and purchased components on the
basis of their cost. This is a separate charge in addition to manufacturing overhead. Leis‟ annual manufacturing overhead budget is one-third variable and
two-thirds fixed. Garland Company, one of Leis‟ reliable vendors, has offered to supply part WS73 at a unit price of P15,000.
If Leis purchases the WS73 units from Garland, the capacity being used by Leis to manufacture these parts would be idle. Should Leis decide to
purchase the parts from Garland, the unit cost of WS73 would
A. Increase by P4,800 C. Decrease by P6,200
B. Decrease by P3,200 D. Increase by P1,800

xxxviii. The Rural Cooperative, Inc. produces 1,000 units of Part M per
month. The total manufacturing costs of the part are as follows: Direct materials P10,000
Direct labor 5,000
Variable overhead 5,000
Fixed overhead 30,000
Total manufacturing cost P50,000
An outside supplier has offered to supply the part at P30 per unit. It is estimated that 20% of the fixed overhead being assigned to Part M
will no longer be incurred if the company purchases the part from the outside supplier.
If Rural Cooperative purchases 1,000 units of Part M from the outside supplier, its monthly operating income will
A. decrease by P 4,000 C. increase by P 1,000
B. decrease by P20,000 D. increase by P20,000

xxxix.Migs Corporation currently manufactures all component parts used in the manufacture of various hand tools. A steel handle is used in three different
tools. The budgeted costs per unit based on 20,000 units are:
Direct material P6.00
Direct labor 4.00
Variable overhead 1.00
Fixed overhead 2.00
Total unit cost P13.00
Sans Steel, Inc. has offered to supply 20,000 units of the handle to Migs for P12.50 each delivered. If Migs currently has idle capacity that cannot
be used, accepting the offer will
A. Decrease the handle unit cost by P0.50.
B. Increase the handle unit cost by P1.50.
C. Decrease the handle unit cost by P1.50.
D. Increase the handle unit cost by P0.50.

xl. The Minolta, Inc. produces 1,000 units of Part M per month. The total manufacturing costs of the part are as
follows: Direct materials P10,000
Direct labor 5,000
Variable overhead 5,000
Fixed overhead 30,000
Total manufacturing cost P50,000
An outside supplier has offered to supply the part at P30 per unit. It is estimated that 20% of the fixed overhead assigned to Part M will no longer
be incurred if the company purchases the part from the outside supplier.
If Minolta purchases 1,000 units of Part M from the outside supplier per month, then its monthly operating income will
A. decrease by P 4,000 C. increase by P 1,000
B. decrease by P20,000 D. increase by P20,000

xli. Bulacan Company manufactures part G for use in the production of its principal product. The costs per unit for 10,000 units of part G are as
follows: Direct materials P 3
Direct labor 15
Variable overhead 6
Fixed overhead 8
Total P32
Pampanga Company has offered to sell Bulacan 10,000 units of part G for P30 per unit. If Bulacan accepts Pampanga‟s offer, the released facilities could
be used to save P45,000 in relevant costs in the manufacture of part H. In addition, P5 per unit of the fixed overhead applied to part G would
continue.
What alternative is more desirable and by what amount?
A. B. C. D.
Alternative Manufacture Manufacture Buy Buy
Amount P10,000 P15,000 P15,000 P10,000

xlii. Blade Division of Dana Company produces hardened steel blades. One-third of the Blades Division‟s output is sold to the Lawn Products Division of Dana;
the remainder is sold to outside customers. The Blade Division‟s estimated sales and standard costs data for the fiscal year ending June 30 are as
follows:
Lawn Products Outsiders
Sales P15,000 P40,000
Variable costs (10,000) (20,000)
Fixed costs (3,000) (6,000)
Gross margin P 2,000 14,000
Unit sales 10,000 20,000
The Lawn Products Division has an opportunity to purchase 10,000 identical quality blades from an outside supplier at a cost of P1.25 per unit on a
continuing basis. Assume that the Blade Division cannot sell any additional products to outside customers.
Should Dana allow its Lawn Products Division to purchase the blades from the outside supplier, and why?
A. Yes, because buying the blades would save Dana Company P500.
B. No, because making the blades would save Dana Company P1,500.
C. Yes, because buying the blades would save Dana Company P2,500.
D. No, because making the blades would save Dana Company P2,500

xliii. The Connell Company uses 5,000 units of Part 501 each year. The cost of manufacturing one unit Part 501 at this volume is as
follows: Direct materials P2.50
Direct labor 3.50
Variable overhead 1.50
Fixed overhead 1.00
Total P8.50
An outside supplier has offered to sell Connell unlimited quantities of Part 501 at a unit cost of P7.75. If Connell accepts this offer, it can eliminate 50
percent of the fixed costs assigned to part 501. Furthermore, the space devoted to the manufacture of Part 501 would be rented to another company for
P6,000 per year. If Connell accepts the offer of the outside supplier, annual profits will
A. Increase by P13,500 C. Increase by P 7,250
B. Increase by P11,000 D. Increase by P 1,250

Point of indifference - Units


xliv. Mars Industries is a multi-product company that currently manufactures 30,000 units of Part QS42 each month for use in the production of its main
product. The facilities now being used to produce Part QS42 have fixed monthly cost of P150,000 and a capacity to produce 84,000 units per
month. If Mars were to buy Part QS42 from an outside supplier, the facilities would be idle, but 60 percent of its fixed costs would not continue. The
variable production costs of Part QS42 are P11 per unit.
If Mars Industries is able to obtain Part QS42 from an outside supplier at a unit purchase price of P12.875, the monthly usage at which it will be
indifferent between purchasing and making Part QS42 is
A. 30,000 units C. 80,000 units
B. 32,000 units D. 48,000 units

Point of indifference - price


xlv.Calero Manufacturing Company can make 100 units of a necessary component part with the following
costs: Direct Materials P80,000
Direct Labor 13,000
Variable Overhead 40,000
Fixed Overhead 27,000
If Calero Manufacturing Company purchases the component externally, P20,000 of the fixed costs can be avoided. At what external price for the 100 units
is the company indifferent between making or buying?
A. P160,000. C. P153,000.
B. P113,000. D. P133,000.

Profit maximization
Point of indifference
xlvi. Dipsum Soft Drinks makes three products: iced tea, soda, and lemonade. The following data are available:
Iced Tea Soda Lemonade
Sales price per unit P9.00 P6.00 P5.00
Variable cost per unit 3.00 1.50 1.00
Contribution margin per unit P6.00 P4.50 P4.00
Dipsum is experiencing a bottleneckn one of s s that affectseach product as follows:
i it processe Lemonade
Iced Soda
Tea
Bottleneck process hours
3 3 4
Whatper unitfor lemonade would equate its profitability to that of
price
soda? A. P8.00. C. P6.00.
B. P7.00. D. P5.50.

Optimal mix
xlvii. Product A sells for P12 per unit and its variable cost per unit is P10. Product B sells for P15 per unit and its variable cost per unit is P12. The
plant capacity is 350,000 machine hours and both products require one machine hour to manufacturer. Which of the following will provide the best sales
mix of Product A and Product B assuming the market limitation of Product A is 200,000 units and the market limitation of Product B is 250,000
units?
A. 250,000 units of Product A, 100,000 units of Product B
B. 50,000 units of Product A, 300,000 units of Product B
C. 100,000 units of Product A, 250,000 units of Product B
D. 150,000 units of Product A, 200,000 units of Product B

xlviii. The Hingis Corporation manufactures two products: X and Y. Contribution margin per unit is determined as follows:
Product X Product Y
Revenue P130 P80
Variable costs 70 P38
Contribution P 60 P42
margin
Total demand for X is 16,000 units and for Y is 8,000 units. Machine hour is a scarce resource. 42,000 machine hours are available during the
year. Product X requires 6 machine hours per unit while product Y requires 3 machine hours per unit.
How many units of X and Y should Hingis Corporation produce?
A. B. C. D.
Product X 16,000 8,000 7,000 3,000
Product Y zero 4,000 zero 8,000

xlix. Mary Manufacturing has assembled the following data pertaining to two popular products.
Blender Electric
mixer
Direct materials P6 P11
Direct labor 4 9
Factory overhead @ P16 per hour 16 32
Cost if purchased from an 20 38
outside supplier
Annual demand (units) 20,000 28,000
Past experience has shown that the fixed manufacturing overhead component included in the cost per machine hour averages P10. Mary has a policy of
filling all sales orders, even if it means purchasing units from outside suppliers.
If 50,000 machine hours are available, and Mary Manufacturing desires to follow an optimal strategy, it should
A. produce 25,000 electric mixers, and purchase all other units as needed
B. produce 20,000 blenders and 15,000 electric mixers, and purchase all other units as needed
C. produce 20,000 blenders and purchase all other units as needed
D. produce 28,000 electric mixers and purchase all other units as needed

Decision
l. A company can sell all the units it can produce of either Product A or Product B but not both. Product A has a unit contribution margin of P36 and
takes two machine hours to make and Product B has a unit contribution margin of P45 and takes three machine hours to make. If there are 1,000
machine hours available to manufacture a product, income will be
A. P3,000 more if Product A is made. C. P3,000 less if Product A is made.
B. P3,000 less if Product B is made. D. the same if either product is made.

li. The Baco Company produces three products with the following costs and selling prices:
A B C
Selling price per unit P16 P21 P21
Variable cost per unit 7 11 13
Contribution margin per unit P 9 P10 P8
Direct labor hours per unit 1.0 1.5 2.0
Machine hours per unit 4.5 2.0 2.5
In what order should the three products be produced if either the direct labor-hours or the machine hours are the company‟s production
constraint?
A. B. C. D.
Direct labor A, B, C B, C, A B, C, A A, B, C
hours
Machine hours B, C, A B. C. A A, C, B A, C, B

lii. Scarce Company has been producing two types of bearings, Plastic and Metal, for its own use in the production of main products. The data
regarding these two bearings follow:
Plastic Metal
Machine hours required per 3.0 4.5
unit
Standard cost per unit
Prime costs P 8.00 P 9.00
Variable overhead* 3.00 4.00
Fixed overhead** 4.50 6.75
Total P15.50 P19.75

*Variable manufacturing overhead is applied on the basis of direct labor hours.


**Fixed manufacturing overhead is applied on the basis of machine hours.

Scarce‟s annual requirements for these bearings is 7,000 units of Plastic and 11,000 units of Metal. Recently, Scarce‟s management decided to
devote additional machine hours to other product lines resulting to only 48,000 machine hours per year that can be dedicated to the production of
the bearings. An outside company has offered to sell Scarce the annual supply of the bearings at prices of P15.50 for Plastic and P17.50 for Metal.
Scarce wants to schedule the otherwise idle 48,000 machine hours to produce bearings so that the company can minimize its costs (maximize its
net benefits).
Scarce Company will maximize its net benefits by purchasisng
A. 7,000 units of Plastic and manufacturing the remaining bearings.
B. 11,000 units of Metal and manufacturing 7,000 units of Plastic.
C. 6,000 units of Plastic and manufacturing the remaining bearings.
D. 5,000 units of Metal and manufacturing the remaining bearings.

liii. HILO Company manufactures electric carpentry tools. The production department had met all production requirements for the current month and
has an opportunity to produce additional units of product with its excess capacity. Unit selling prices and unit costs for three different drill models
are as follows:
Home Deluxe Pro Model
Model Model
Selling price P58 P65 P80
Direct material 16 20 19
Direct labor (P10 per 10 15 20
hour)
Variable overhead 8 12 16
Fixed overhead 16 5 15
Variable overhead is applied on the basis of direct-labor pesos, while fixed overhead is applied on the basis of machine hours. There is sufficient demand
for the additional production of any model in the product line. If it has excess machine capacity but a limited amount of labor time, to which
product or products should HILO Company devote its excess production?
A. Home model C. Deluxe model
B. Pro Model D. Equally

liv. Product A sells for P12 per unit and its variable cost per unit is P10. Product B sells for P15 per unit and its variable cost per unit is P12. The
plant capacity is 350,000 machine hours and Product A requires 48 minutes to complete while Product B requires 75 minutes. Which of the following will
provide the best sales mix of Product A and Product B assuming the market limitation of Product A is 200,000 units and the market limitation of
Product B is 250,000 units?
A. 46,875 units of Product A, 250,000 units of Product B
B. 200,000 units of Product A, 152,000 units of Product B
C. 152,000 units of Product A, 200,000 units of Product B
D. 100,000 units of Product A, 250,000 units of Product B

lv. Dimasalang Company has only 25,000 hours of machine time each month to manufacture its two products. Product X has a contribution margin of P50
and Product Y has a contribution margin of P64. Product X requires 5 machine hours and Product Y, 8 hours. If Dimasalang wants to dedicate 80% of its
machine time to the product that will provide the most income, it will have a total contribution margin of
A. P250,000 C. P210,000
B. P240,000 D. P200,000

Sell-as-is-or-process-further
Minimum sales
lvi. Snow Clean Corporation produces cleaning compounds and solutions for industrial and household use. While most of its products are processed
independently, a few are related. Grit 337, a coarse cleaning powder with many industrial uses, costs P16 a pound to make and sells for P20 a pound. A
small portion of the annual production of this product is retained for further processing in the Mixing Department, where it is combined with several other
ingredients to form a paste, which is marketed as a silver polish selling for P40 per jar. This further processing requires ¼ pound of Grit 337 per
jar. Costs of other ingredients, labor, and variable overhead associated with this further processing amount to P25 per jar. Variable selling costs are P3
per jar. If the decision were made to cease production of the silver polish, P56,000 of Mixing Department fixed costs could be avoided. Snow Clean has
limited production capacity for Grit 337, but unlimited demand for the cleaning powder.
What is the minimum number of jars of silver polish that would have to be sold to justify further processing of Grit
337. A. 8,000 C. 7,000
B. 5,600 D. 4,667

Decision
lvii. Beal Company is starting business and is unsure of whether to sell its product assembled or unassembled. The unit cost of the unassembled product is
P40 and Beal Company would sell it for P90. The cost to assemble the product is estimated at P18 per unit and Beal Company believes the market would
support a price of P116 on the assembled unit.
What is the correct decision using the sell or process further decision rule?
A. Sell before assembly, the company will be better off by P18 per unit.
B. Sell before assembly, the company will be better off by P26 per unit.
C. Process further, the company will be better off by P26 per unit.
D. Process further, the company will be better off by P8 per unit.

lviii. Sales of 25,000 units at P7.20 per unit are made monthly. The unit cost is P5.90. Incremental costs of P1.35 per unit to further process the units will
result in the 25,000 units being sold for P8.75 each. Which course of action should the company take?
A. Commit its resources to a different product
B. Sell the units at the current stage of completion
C. Do further processing and sell the units at P8.75
D. Do further processing on only one-half of the units

lix. Aaron Company produces a product that can be sold for P250,000 at an intermediate stage. If Aaron finishes the product, they will incur P75,000 of
additional material costs and another P15,000 in labor and overhead costs. When finished, Aaron will be able to sell the product for P350,000.
Which of the following answers is correct?
A. Sell now
B. Finish the product because profits will increase by P25,000
C. Finish the product because profits will increase by P12,500
D. Finish the product because profits will increase by P10,000

Effect of decision
lx. Ottawa Corporation produces two products from a joint process. Information about the two joint products follows:
Product X Product Y
Anticipated production 2,000 lbs 4,000 lbs
Selling price per pound at split-off P30 P16
Additional processing costs/pound after P15 P30
split-off (all variable)
Selling price/pound after P40 P50
further processing
The joint cost is P85,000. Ottawa currently sells both products at the split-off point. If Ottawa makes decision which maximizes profit, its profit will
increase by A. P16,000 C. P 4,000
B. P50,000 D. P10,000

lxi. The cost to manufacture an unfinished unit is P40 (P30 variable and P10 fixed). The selling price per unit is P50. The company has unused
production capacity and has determined that units could be finished and sold for P65 with an increase in variable costs of 40%. What is the additional net
income per unit to be gained by finishing the unit?
A. P 3 C. P10
B. P15 D. P12
Total processing cost
lxii. Matador Manufacturing schedules a weekly production of 15,000 units of Product M and 30,000 units of N for which P800,000 common variable costs are
incurred. These two products can be sold as is or processed further. Further processing of either product does not delay the production of subsequent
batches of the joint products. Below are some of the information:
M N
Unit selling price without further processing P25 P19
Unit selling price with further processing P31 P23
Total separate weekly variable costs of further processing P100,000 P110,000
To maximize Matador‟s manufacturing contribution margin, the total separate variable costs of further processing that should be incurred each
week are A. P105,000 C. P110,000
B. P100,000 D. P210,000

Keep-or-drop decision
Analysis
lxiii. A company is deciding whether or not to eliminate a segment of its business. The segment generates total sales of P104,000, its direct
expenses are P22,000, and its indirect expenses are P26,000. Its cost of goods sold is P64,000. Six thousand pesos of the direct expenses and
P8,000 of its indirect expenses are avoidable expenses. Which of the following is not true?
A. This segment has a net loss of P8,000.
B. This segment's revenue is greater than its avoidable costs.
C. This segment is a good candidate for elimination.
D. This segment's avoidable costs are greater than unavoidable costs.

Effect of drop decision


lxiv. Banahaw Company plans to discontinue a department that has a contribution margin of P240,000 and P480,000 in fixed costs. Of the fixed
costs, P210,000 can be avoided. The effect of this discontinuance on Banahaw‟s overall net operating income would be a(an)
A. decrease of P30,000 C. increase of P30,000
B. decrease of P10,000 D. increase of P10,000

lxv.Mina Co. mines three products. Gold Ore sells for P1,000,000 per ton, variable costs are P600,000 per ton, and fixed mining costs are
P6,000,000. The segment margin for 2007 was P1,200,000. The management of Mina Co. was considering dropping the mining of Gold
Ore. Only one-half of the fixed
expenses are direct and would be eliminated if the segment was dropped. If Gold Ore were dropped, net income for Mina Co. would
A. Increase by P2,000,000 C. Decrease by P2,000,000
B. Increase by P1,200,000 D. Decrease by P1,200,000

lxvi. Agimat Company plans to discontinue a segment with a P32,000 segment margin. Common expenses allocated to the segment amounted to
P45,000, of which P20,000 cannot be eliminated if the segment were closed. The effect of closing down the segment on Agimat Company‟s
before tax profit would be
A. P12,000 decrease C. P12,000 increase
B. P 7,000 decrease D. P 7,000 increase

Shutdown point
lxvii. Bulusan Company normally produces and sells 30,000 units of E14 each month. E14 is a small electrical relay used in the automotive industry
as a component part in various products. The selling price is P22 per unit, variable costs are P14 per unit, fixed manufacturing overhead costs total
P150,000 per month, and fixed selling costs total P30,000 per month.

Employment-contract strikes in the companies that purchase the bulk of the E14 have caused Bulusan Company‟s sales to temporarily drop to only 9,000
units per month. Bulusan Company estimates that the strikes will last for about two months, after which time sales of E14 should return to normal.
Due to the current low level of sales, however, Bulusan Company is thinking about closing down its own plant during the two months that the
strikes are on. If Bulusan Company does close down its plant, it is estimated that fixed manufacturing overhead costs can be reduced to P105,000 per
month and that fixed selling costs can be reduced by 10%. Start-up costs at the end of the shutdown period would total P8,000. Since Bulusan Company
uses just-in-time production method, no inventories are on hand.

At what level of unit sales for the two-month period should Bulusan Company be indifferent between temporarily closing the plant or keeping it
open? A. 11,000 C. 10,000
B. 24,125 D. 8,000

Equipment replacement
lxviii. MNL Company has an opportunity to acquire a new machine to replace one of its present machines. The new machine would cost P90,000, have a 5-
year life and no estimated salvage value. Variable operating costs would be P100,000 per year. The present machine has a book value of
P50,000 and a remaining life of 5 years. Its disposal value now is P5,000, but it would be zero after 5 years. Variable operating costs would be
P125,000 per year. Ignore income taxes. Considering the 5 years in total, what would be the difference in profit before income taxes by acquiring
the new machine as opposed to
retaining the present one?
A. P10,000 decrease C. P35,000 increase
B. P15,000 decrease D. P40,000 increase

Lease
lxix. Darren Co. is considering disposing an equipment that costs P50,000 and has P40,000 of accumulated depreciation to date. Darren Co. can
sell the equipment through a broker for P25,000 less 5% commission. Alternatively, Minton Co. has offered to lease the equipment for five years for a
total of P48,750. Darren will incur repair, insurance, and property tax expenses estimated at P10,000. At lease-end, the equipment is expected to have no
residual value. The net differential income from the lease alternative is:
A. P15,000. C. P25,000.
B. P 5,000. D. P12,500.

Comprehensive
Questions 70 through 74 are based on the following information:
Adrenal Company has a single product called a CAD. The company normally produces and sells 60,000 CADS each year at a selling price of P32
per unit. The company‟s unit costs at this level of activity are given below:
Direct materials P10.00
Direct labor 4.50
Variable manufacturing overhead 2.30
Fixed manufacturing overhead 5.00
Variable selling expenses 1.20
Fixed selling expenses 3.50
Total cost per unit P26.50

lxx.Assume that Adrenal Company has sufficient capacity to produce 90,000 CADS each year without any increase in fixed manufacturing overhead costs.
The company could increase its sales by 25% above the present 60,000 units each year if it were willing to increase the fixed selling expenses by
P80,000. The increase in income if the production is increased by 25% is
A. P130,000 C. P 25,000
B. P108,333 D. P 20,800
lxxi. Assume again that Adrenal Company has sufficient capacity to produce 90,000 CADS each year. A customer in a foreign market wants to purchase
20,000 CADS. Import duties on the CADS would be P1.70 per unit, and costs for permits and licenses would be P9,000. The only selling costs
that would be associated with the order would be P3.20 per unit shipping cost. What is the break-even price on this order?
A. P23.35 C. P22.15
B. P28.65 D. P21.70

lxxii. The company has 1,000 CADS on hand that have some deformities and are therefore considered to be “seconds.” Due to the deformities, it
will be impossible to sell these units at the normal price through regular distribution channels. What unit cost figure is relevant for setting a minimum
selling price?
A. P16.80 C. P 4.70
B. P18.00 D. P 1.20

lxxiii. Due to a strike in its supplier‟s plant, Adrenal Company is unable to purchase more material for the production of CADS. The strike is expected
to last for two months. Adrenal Company has enough material on hand to continue to operate at 30% of normal levels for the two months. If the plant
were closed, fixed overhead costs would continue at 60% of their normal level during the two-month period; the fixed selling costs would be reduced by
20% while the plant was closed. How much is the advantage or disadvantage of closing the plant for the two-month period?
A. Disadvantage, P144,000 C. Disadvantage, P15,000
B. Advantage, P144,000 D. Advantage, P15,000

lxxiv. An outside manufacturer has offered to produce CADS for Adrenal Company and to ship them directly to Adrenal‟s customers. If Adrenal Company
accepts this offer, the facilities that it uses to produce CADS would be idle; however, fixed overhead costs would be reduced by 75% of their
present level. Since the outside manufacturer would pay for all the costs of shipping, the variable selling costs would be only two-thirds of their present
amount. What is the unit cost figure that is relevant for comparison to whatever quoted price is received from the outside manufacturer?
A. P20.95 C. P20.55
B. P21.35 D. P16.80

Question Numbers 75 though 77 are based on the following:


Henderson Equipment Company has produced a pilot run of 50 units of a recently developed cylinder used in its finished products. The company
expects to produce and sell 800 units. The pilot run required 14.25 direct-labor hours for the 50 cylinders, averaging 0.285 direct-labor hours per cylinder.
Henderson has experienced a significant learning curve on the direct-labor hours needed to produce new cylinders. As cumulative output doubles,
say from 25 to 50 units for
example, the average labor time per unit declines by 20 percent. Past experience indicates that learning tends to cease by the time 800 parts are
produced. Henderson‟s manufacturing costs for cylinders are as follows:
Direct labor P120.00 per hour
Variable overhead 100.00 per direct labor hour
Fixed overhead 166.00 per direct labor hour
Direct material 40.50 per unit

Henderson has received a quote of P75 per unit from the Leyte Machine Company for the additional 750 cylinders needed. Henderson frequently
subcontracts this type of work and has always been satisfied with the quality of the units produced by Leyte. Recently, Henderson Equipment Company has
been operating at considerably less than full capacity.

lxxv. How many direct-labor hours are expected to be used for the production of 800 cylinders (including the pilot run)?
A. 93.4 hours C. 79.1 hours
B. 74.7 hours D. 67.6 hours

lxxvi. The production of 800 cylinders, including the pilot run, requires total incremental
costs of: A. P48,834 C. P68,452
B. P49,802 D. P52,948

lxxvii. The effect on profit of producing 750 units instead of buying them from Leyte Machine Company a(an)?
A. Increase of P 8,470. C. Increase of P12,676.
B. Increase of P 7,052. D. Decrease of P22,560.

Questions 78 through 81 are based on the following:


CLASP Industries received an order for a piece of special machinery from Tigok Company. Just as CLASP completed the machine, Tigok declared
backruptcy, defaulted on the order, and forfeited the 10 percent deposit paid on the selling price of P72,500.

CLASP‟s manufacturing manager identified the costs already incurred in the production of the special machinery for Tigok as

follows: Direct material P16,600


Direct labor 21,400
Manufacturing overhead:
Variable P10,700
Fixed 5,350 16,050
Fixed selling and administrative costs 5,405
Total P59,455

Another company, Kay Corporation, will buy the special machinery if it is reworked to Kay‟s specifications. CLASP offered to sell the reworked machinery to
Kay as a special order for P68,400. Kay agreed to pay the price when it takes delivery in two months. The additional identifiable costs to rework the
machinery to Kay‟s specifications are as follows:

Direct materials P 6,200


Direct labor 4,200
Total P10,400

A second alternative available to CLASP is to convert the special machinery to the standard model, which sells for P62,500. The additional identifiable costs
for this conversion are as follows:

Direct materials P2,850


Direct labor 3,300
Total P6,150

A third alternative for CLASP is to sell the machine as is for a price of P52,000. However, the potential buyer of the unmodified machine does not
want it for 60 days. This buyer has offered a P7,000 down payment, with the remainder due upon delivery.

The following additional information is available regarding CLASP‟s operations:

1. The sales commission rate on sales of standard models is 2 percent, while the rate on special orders is 3 percent.

2. Normal credit terms for sales of standard models are 2/10, net/30. This means that a customer receives a 2 percent discount if payment is made
within 10
days, and payment is due no later than 30 days after billing. Most customers take the 2 percent discount. Credit terms for a special order are negotiated
with the customer.

3. The allocation rates for manufacturing overhead and fixed selling and administrative costs are as
follows: Manufacturing costs:
Variable 50% of direct-labor costs
Fixed 25% of direct-labor costs
Fixed selling and administrative costs 10% of the total manufacturing costs

4. Normal time required for rework is one month.

lxxviii. How much peso contribution would the sale to Kay Corporation add to CLASP‟ before-tax profit?
A. P53,848 C. P55,900
B. P55,948 D. P 9,300

lxxix. How much peso contribution would the alternative of converting the special machinery to standard model add to CLASP‟s before-tax profit?
A. P52,200 C. P52,825
B. P54,475 D. P 7,650

lxxx. If Kay makes CLASP a counteroffer, what is the lowest price CLASP should accept for the reworked machinery from Kay?
A. P10,400 C. P10,722
B. P12,500 D. P12,887

lxxxi. How much would the alternative of selling unmodified machinery to the potential buyer contribute to CLASP‟s before-tax profit?
A. P50,440 C. P49,920
B. P 1,740 D. P49,400

Question Nos. 82 and 85 are based on the following:


Constraint Company manufactures and sells three products, which are manufactured in a factory with four departments. Both labor and machine time are
applied to the products as they pass through each department. The machines and labor skills required in each department are so specialized that neither
machines nor
labor can be switched from one department to another.

Constraint Company‟s management is planning its production schedule for the next few months. The planning is complicated, because there are labor
shortages in the community and some machines will be down several months for repairs.

Management has assembled the following information regarding available machine and labor time by department and the machine hours and direct-labors
required per unit of product. These data should be valid for the next six months.

DEPARTMENT
Monthly Capacity Available 1 2 3 4
Norman machine capacity in 3,500 3,500 3,000 3,500
MH
Capacity of machines being
repaired in machine hours ( 500) ( 400) ( 300) ( 200)
Available machine capacity in 3,000 3,100 2,700 3,300
MH
Available direct labor hours 3,700 4,500 2,750 2,600
(DLH)

Labor and Machine


Specifications per Unit of
Product
Product Labor
and Machine Time
401 Direct labor 2 3 3 1
hours
Machine hours 1 1 2 2
403 Direct labor 1 2 - 2
hours
Machine hours 1 1 - 2
405 Direct labor 2 2 2 1
hours
Machine hours 2 2 1 1

The sales department believes that the monthly demand for the next six months will be as follows:

Product Monthly Unit Sales


401 500
403 400
405 1,000

Inventory levels are satisfactory and need not be increased or decreased during the next six months. Unit price and cost data that will be valid for the
next six months are as follows:

PRODUCTS
401 403 405
Unit costs:
Direct material P 7 P 13 P 17
Direct labor
Department 1 12 6 12
Department 2 21 14 14
Department 3 24 -- 16
Department 4 9 18 9
Variable overhead 27 20 25
Fixed overhead 15 10 32
Variable selling expenses 3 2 4
Unit selling price P196 P123 P167

lxxxii. Which department has capacity constraint in labor hours?


A. Department 1 C. Department 3
B. Department 2 D. Department 4

lxxxiii. The total Machine Hours required by estimated monthly unit sales
are: A. 10,600 C. 11,600
B. 12,100 D. 13,500

lxxxiv. The total number of labor hours as constraint for a month


is: A. 50 C. 300
B. 750 D. No constraint

lxxxv. In order to maximize its monthly profit, Constraint Company should prod uce:
A. B. C. D.
401 250 250 500 500
403 0 400 400 0
405 1,000 1,000 625 625

Question Nos. 86 through 89 are based on the following;


Arnold Syjuco operates a small machine shops. He manufactures one standard product available from many other similar businesses and he also
manufactures products to customer order. Hi accountant prepared the annual income statement shown below:

Custom Sales Standard Sales Total


Sales P1,000,000 P500,000 P1,500,000
Material P 200,000 P160,000 P 360,000
Labor 400,000 180,000 580,000
Depreciation 126,000 72,000 198,000
Power 14,000 8,000 22,000
Rent 120,000 20,000 140,000
Heat and light 12,000 2,000 14,000
Other 8,000 18,000 26,000
Total P 880,000 P460,000 P1,340,000
Income P 120,000 P 40,000 P 160,000

The depreciation charges are for machines used in the respective product lines. The power charge is apportioned on the estimate of power consumed. The
rent is for the building space which has been leased for 10 years at P140,000 per year. The rent and heat and light are apportioned to the product lines
based on amount of floor space occupied. All other costs are current expenses identified with the product line incurring them.

A valued custom parts customer has asked Mr. Syjuco to manufacture 5,000 special units for him. Mr. Syjuco is working at capacity and would have
to give up some other business to take this business. He cannot renege on custom orders already agreed to but he could reduce the output of his standard
product by about one-half for one year while producing the specially requested custom part. The customer is willing to pay P140 for each part. The material
cost will be about P40 per unit and the labor will be P72 per unit. Mr. Syjuco will have to spend P40,000 for a special device which will be discarded
when the job is done.

lxxxvi. What is the incremental cost of the special order of 5,000 units?
A. P600,000 C. P779,000
B. P421,000 D. P371,000

lxxxvii. What is the full cost of the special order?


A. P779,000 C. P421,000
B. P492,400 D. P651,000

lxxxviii. The amount of opportunity cost of taking the special order


is: A. P183,000 C. P250,000
B. P 71,000 D. P124,600

lxxxix. What is the effect on the overall profit if the special order is accepted?
A. P450,000 C. P( 25,000)
B. P( 85,000) D. P 29,000

Question Nos. 90 through 94 are based on the following:


The Verbatim Corporation, which produces and sells to wholesalers a highly successful line of summer lotions and insect repellents, has decided to diversify
in
order to stabilize sales over the year. A natural area for the company to consider is the production of special lotion and cream to prevent dry and
chapped skin.

After considerable research, a special product line has been developed. However, because of the conservatism of the company management,
Verbatim‟s president has decided to introduce only one of the new products for this coming rainy season. If the product is a success, further
expansion will be initiated in future years.

The product selected (called Chaps) is a lip balm that will be sold in a lipstick-type tube. The product will be sold to wholesalers in boxes of 24 tubes
for P800 per box. Because of available capacity, no additional fixed charges will be incurred to produce the product. However, a P10,000,000 fixed charge
will be absorbed by the new product to allocate a fair share of the company‟s present fixed costs to it.

Using the estimated sales and production of 100,000 boxes of Chaps as the standard volume, the accounting department has developed the following

costs: Direct labor P200 per box


Direct materials 300 per box
Total overhead 150 per box
Total P650 per box

Verbatim has approached a cosmetics manufacturer to discuss the possibility of purchasing the tubes for Chaps. The purchase price of the empty tubes from
the cosmetics manufacturer would be P90 per 24 tubes. If the Verbatim Corporation accepts the purchase proposal, it is estimated that direct labor
and variable overhead costs would be reduced by 10% and direct material costs would be reduced by 20%.

xc. What is the variable overhead rate per box of Chaps?


A. P100 C. P 50
B. P150 D. P200

xci.What is the material cost per box of Chaps saved by purchasing them?
A. P300 C. P 60
B. P240 D. P 30
xcii. How much would it cost Verbatim to produce the tubes per box?
A. P 60 C. P 90
B. P 85 D. P120

xciii. How much would Verbatim incur by making 125,000 boxes, assuming that additional equipment, at an annual rental of P1,000,000, must be acquired
to produce this volume?
A. P10,625,000 C. P11,250,000
B. P11,625,000 D. P12,500,000

xciv. Referring to Question No. 93, what is the impact on its profit if Verbatim were to buy 125,000 boxes?
A. Additional profit of P1,000,000. C. Additional profit of P375,000.
B. Additional profit of P1,250,000. D. Decrease in profit of P625,000.

Question Nos. 95 through 101 are based on the following:


Medical Supply Company produced hydraulic hoists that were used by hospitals to move bedridden patients. The costs of manufacturing and marketing
hydraulic hoists at the company‟s normal volume of 3,000 units per month are show below:

Unit manufacturing costs:


Direct materials P1,000
Direct labor 1,500
Variable overhead 500
Fixed overhead 1,200 P4,200
Unit marketing costs:
Variable 500
Fixed 1,400 1,900
Total unit costs P6,100

Unless otherwise stated, assume there is no connection between the situations described in the questions; each is to be treated independently. Unless
otherwise stated, a regular selling price of P7,400 per unit should be assumed. Ignore income taxes and other costs that are not mentioned in the
cost schedule or in a question itself.
xcv. What is the monthly breakeven units for Medical Supply Company?
A. 2,000 C. 1,950
B. 2,689 D. 2,614

xcvi. Market research estimates that volume could be increased to 3,500 units, which is well within hoist production capacity limitations, if the price were ct
from P7,400 to P6,500 per unit. Assuming the cost behavior patterns implied by the data in the cost schedule is correct, would you recommend
this action be taken?
A. Yes, because the profit will increase by P1,500,000.
B. Yes, because the profit will increase by P 200,000.
C. No, because the profit will decrease by P1,200,000.
D. No, because the profit will decrease by P2,400,000.

xcvii. On March 1, a contract offer is made to Medical Supply Company by the Veterans‟ Hospital to supply 500 units for delivery by March 31.
Because of an unusually large number of rush orders form their regular customers. Medical Supply plans to produce 4,000 units during March, which will
use all available capacity. If the Veterans‟ Hospital‟s order is accepted, 500 units normally sold to regular customers would be lost to a competitor. The
contract given by the hospital would reimburse the Veterans‟ Hospital‟s share of March manufacturing costs, plus pay a fixed fee (profit) of P500,000.
(There would be no variable marketing costs incurred on the hospital‟s unit.) What impact would accepting the Veterans‟ Hospital contract have on
March income?
A. P 1,100,000 C. P(1,350,000)
B. P( 850,000) D. P 500,000

xcviii. Medical Supply Company has an opportunity to enter a foreign market in which price competition is keen. An attraction of the foreign market is that
demand there is greatest when demand in the domestic market is quite low; thus idle production facilities could be used without affecting
domestic business.
An order for 1,000 units is being sought at a below-normal price in order to enter this market. Shipping costs for this order will amount to P750
per unit, while total costs of obtaining the contract (marketing costs) will be P40,000. No other variable marketing costs would be required on this order.
Domestic business would be unaffected by this order. What is the minimum unit price should Medical Supply Company consider for this order of
1,000 units?
A. P3,750 C. P3,790
B. P3,000 D. P4,290
xcix. An inventory of 230 units of an obsolete model of the hoist remains in the stockroom. These must be sold through regular channels at reduced
prices, or the inventory will soon be valueless. What is the minimum price that would be acceptable in selling these units?
A. P3,500 C. P3,000
B. P4,200 D. P 500

c. A proposal is received from an outside contractor who will make and ship 1,000 hydraulic hoist units per month directly to Medical Supply‟s
customers as orders are received from Medical Supply‟s sales staff. Medical Supply‟s fixed marketing costs would be unaffected, but its variable
marketing costs would be cut by 20 percent for these 1,000 units produced by the contractor. Medical Supply‟s plant would operate at two thirds of its
normal level, and total allocated fixed manufacturing costs for these 1,000 units would be cut by 30 percent. What in-house unit cost should be
used to compare with the quotation received from the supplier?
A. P 3,760 C. P 4,240
B. P 3,000 D. P 3,460

ci. Assume the same facts as in requirement No. 101 except that the idle facilities would be used to produce 800 modified hydraulic hoists per month
for us in hospital operating rooms. These modified hoists could be sold for P9,000 each, while the costs of production would be P5,500 per unit variable
manufacturing expense. Variable marketing costs would be P1,000 per unit. Fixed marketing and manufacturing costs would be unchanged
whether the original 3,000 regular units hoists were manufactured or the mix of 2,000 regular hoists plus 800 modified hoists were produced. What is
the maximum purchase price per unit that Medical Supply should be willing to pay the outside contractor?
A. P 5,100 C. P 5,500
B. P 3,100 D. P 5,600

Question Nos. 102 and 103 are based on the following:


Marcus Fibers, Inc., specializes in the manufacturing of synthetic fibers that the company uses in many products such as blankets, coats, and uniforms for
police and firefighters. Marcus has been in business since 1975 and has been profitable every year since 1983. The company uses a standard cost system
and applies overhead on the basis of direct labor hours.

Marcus has recently received a request to bid on the manufacture of 800,000 blankets scheduled for delivery to several military bases. The bid must be
started at full cost per unit plus a return on full cost of no more than 9 percent after income taxes. Full cost has been defined as including all variable costs of
manufacturing the product, a reasonable amount of fixed overhead, and reasonable incremental administrative costs associated with the manufacture and
sale of the product. The contractor has indicated that bids in excess of P25 per blankets are not likely to be considered.

In order to prepare the bid for the 800,000 blankets, Andrea Lighter, cost accountant, has gathered the following information about the cost associated with the
production of the blankets.

Direct material P 1.50 per pound of fibers


Direct labor P 7.00 per hour
Direct machine costs* P10,00 per blanket
Variable overhead P 3.00 per direct labor hour
Fixed overhead P 8.00 per direct labor
hour Incremental administrative costs P2,500 per 1,000
blankets Special fee** P 0.50 per blanket
Material usage 6 pounds per blanket
Production rate 4 blankets per DLH
Effective tax rate 40%

cii. The minimum price per blanket that Marcus Fibers, Inc., could bid without reducing the company‟s net
income is A. P24.00 C. P50.25
B. P21.50 D. P40.25

ciii. Using the full-cost criteria and the maximum allowable return specified, Marcus Fibers‟ bid price per blanket
would be: A. P24.00 C. P26.00
B. P29.90 D. P27.90

ANSWER EXPLANATIONS
i. Answer: C
Cost of alternative selected P800,000
Cost of alternative rejected 650,000
Incremental cost P150,000

ii. Answer: A
The company needs to purchase 55,000 units earlier than their scheduled 5,000-unit monthly purchase. Hence, the average investment for the inventory is (55,000 x P60 ÷ 2) or
P1,650,000. The opportunity cost is P132,000 or (P1,650,000 x 0.08).

iii. Answer: A
Additional revenue after rework (24,000(12 – 4) P192,000
Less Additional cost (24,000 x 2) 48,000
Additional profit P144,000

iv. Answer: B
The only relevant out-of pocket cost is the variable selling expense which is P40. The sale thru the regular channels involves an opportunity cost of P140.
Variable selling expense (40% x 100) 40
Opportunity cost 140
Total 180

v. Answer: C
Regular variable cost P8.00
Overtime premium 1.50
Relevant cost per unit P9.50

vi. Answer: B
Full cost 50.00
Fixed overhead (180,000/9,000) 20.00
Relevant unit cost 30.00

vii. Answer: C
Cost of 1,000 kg at latest price (1,000 x 8.70) 8,700
Add excess price include on the remaining 4,000 kg. 4,000 x (8.70 – 8.30)
1,600
Relevant cost 10,300

viii.Answer: B
Direct materials (2,000 @ 10) 20,000
Direct labor (2,000 @ 12) 24,000
Variable overhead (2,000 @ 6) 12,000
Increase in variable cost due to overtime (2,000 @ 14) 28,000
Incremental cost 84,000

ix. Answer: B
Variable costs P56,250
Additional fixed costs 13,750
Minimum bid price P70,000

x. Answer: B
Direct material (360,000 ÷ 24,000) P15.00
Direct labor (540,000 ÷ 24,000) 22.50
Variable selling expenses (84,000 ÷ 24,000) 3.50
Total P41.00
Add Profit per unit (22,500 ÷ 1,500) 15.00
Selling price P56.00

xi. Answer: B
Relevant cost to make and sell:
Direct materials 39
Direct labor 6
Variable OH 8
Reduced selling expenses (30 x 0.06) 18
Add‟l fixed cost (20,000 † 5,000) 4
Minimum selling price 75

xii. Answer: B
The company has no existing capacity. The minimum selling price for this special sales should equal the regular selling price plus additional expenses.
Regular selling price P18
Additional expenses 1
Minimum selling price P19

xiii.Answer: A
Direct materials 20.00
Direct labor 15.00
Variable overhead 12.00
Variable shipping and handling 3.00
Lost contribution margin – LB46 (10,000 ÷ 1,000) 10.00
Minimum price 60.00
The lost contribution margin on regular sale is relevant because the company is operating at capacity. In a special sale wherein the company has to give up some of its regular units, the relevant
costs consist of incremental costs plus any opportunity costs.

xiv. Answer: D
Direct materials 4.50
Direct labor 10.00
Variable overhead 3.00
Variable selling expense 1.00
Additional profit (40,000/5,000) 8.00
Required selling price 26.50
xv. Answer: C
The maximum number of units in regular sales that Benjing could afford to lose equals the quantity that provides regular contribution margin that matches the contribution margin provided by special
sale.
Contribution margin from special sale 1,000 (14 – 8) 6,000
Divided by regularCM (20 – 8) ÷ 12
Maximum Number of units 500

To illustrate the solution:


Contribution margin from special sale 6,000
Less Decrease in regular sales‟ contribution margin (500 x 12) 6,000
Effect on profit NIL

xvi. Answer: B
The maximum decrease in regular sale = Contribution margin from special sale/Unit contribution margin on regular
sale (400 x 0.20) ÷ (2.00 -1.50) = 160

xvii. Answer: A
Total contribution margin from special sale(15,000 x P5.50) P82,500
Less Additional fixed costs 30,000
Profit from special sale P52,500
Less Decrease in contribution margin on regular
Sale 2,000(P39 – P22.50) 33,000
Additional profit P19,500
Please refer to Solution for Number regarding details of contribution margin per unit.

xviii. Answer: C
Selling price P6.00
Relevant cost per unit:
Regular cost per P7.50
unit
Less: Commission P0.75
Fixed overhead (P3 x 2/3) 2.00 (2.75)
Net amount P4.75
Incremental fixed cost (P150 300) 0.50 5.25
Advantage per unit, Buy P0.75
Number of units 300
Increase in profit P 225

xix. Answer: D
Additional profit: 3,000 x (70 – 50) = 60,000
xx. Answer: A
Special price 32
Relevant cost:
Direct materials 4
Direct labor 12
Variable overhead 6 22
Unit contribution margin 10
Units ordered 2,000
Additional profit 20,000

xxi. Answer: A
Sales 60,000
Less: Variable production cost (1,500 x 30) 45,000
Additional Fixed cost 5,000
Labeling cost (1,500 x 2.50) 3,750 53,750
Profit 6.250

xxii.Answer: B
The minimum selling price should equal the relevant cost to produce and sell a unit of product.
Direct materials P50
Direct labor 20
Variable overhead (P10 x 0.2) 2
Selling expense (P15 x 0.4) 6
Minimum selling price P78

xxiii. Answer: C
The company has no excess capacity to be devoted to the production of additional units for special sale. In a special sale decision where there is no excess capacity, the minimum selling price must
be equal to the market price less any avoidable expenses.
Selling price P100
Less Avoidable selling expense (P15 x 0.4) 6
Minimum selling price P 94

xxiv. Answer: D
The book value of the old equipment is a sunk cost and therefore not a relevant one. Also, the related cost on outstanding note are irrelevant. They are not affected by a decision.

xxv. Answer: D
Relevant Costs
Beta Zeta
Direct materials 4.00 80.00
Direct labor 10.00 47.00
Factory overhead 40% 16.00 8.00
Relevant Unit cost P30.00 135.00

xxvi. Answer: C
Direct material 2.00
Direct labor 2.40
Variable overhead 1.60
Avoidable marketing cost (0.7 x 2.50) 1.75
Relevant cost Make 7.75
The maximum purchase price, if ever the company has to decide buying the product, is P6.75. Any amount higher than P6.75 will necessarily increase the unit cost of the product.

xxvii. Answer: B
Direct materials 64,000
Direct labor 16,000
Variable overhead 8,000
Avoidable fixed overhead 6,000
Total relevant cost to make 94,000

xxviii. Answer: D
Direct materials 64,000
Direct labor 16,000
Variable overhead 8,000
Additional contribution margin 16,000
Total relevant cost to make 104,000

xxix. Answer: B
Variable cost to make parts (30,000 x 12) 360,000
Cost buy (30,000 x 15) 450,000
Cost savings – “Make” 90,000
decision

xxx. Answer: A
Direct materials 80,000
Direct labor 13,000
Variable overhead 40,000
Avoidable fixed overhead 4,000
Relevant cost – make 137,000
Purchase price 145,000
Advantage – Make 8,000

xxxi. Answer: A
Direct materials 64,000
Direct labor 16,000
Variable overhead 8,000
Total variable cost 88,000
Less Purchase cost 104,000
Avoidable fixed cost 16,000
Add unavoidable FC 5,000
Total fixed overhead 21,000

xxxii. Answer: B
Purchase cost (2,000 x P15) P30,000
Relevant cost to make:
Variable cost (2,000 x P16) – 8,000 P24,000
Avoidable fixed cost (8,000 x 0.25) 2,000 26,000
Additional cost – Buy (Decrease in P 4,000
profit)
Alternative computation for relevant cost to
make: Total cost (2,000 x P16) P32,000
Less unavoidable fixed cost (8,000 x 0.75) 6,000
Relevant cost to make P26,000

xxxiii. Answer: C
Cost of purchase (2,000 x P15) P30,000
Relevant cost – make:
Variable cost (2,000 x P16) – P8,000 P24,000
Avoidable fixed cost (P8,000 x 0.25) 2,000
Opportunity cost – rent 6,000 32,000
Cost savings – Buy (increase in profit) P( 2,000)

xxxiv. Answer: C
Relevant costs to make
Direct materials P 90,000
Direct labor 120,000
Variable overhead 15,000
Supervisor‟s salary 30,000
Opportunity costs, rent 65,000
Total 320,000
Relevant cost to buy (15,000 x P20) 300.000
Advantage - Buy P 20,000
If the company would purchase the units, it would save P20,000.

xxxv. Answer: B
Cost of ink cartridges (5,000 x P15) P75,000
Less: Relevant cost to produce (5,000 x P8) 40,000
Additional cost if ink cartridges are purchased P35,000

xxxvi. Answer: B
Direct material (20,000 @ 6) 120,000
Direct labor (20,000 @30) 600,000
Variable overhead (20,000 @ 120 240,000
Avoidable fixed cost (20,000 @ 9) 180,000
Total relevant costs - Make 1,140,000

Purchase cost (20,000 @ 60) 1,200,000


Add net savings 25,000
Total 1,225,000
Less: Cost to make 1,140,000
Opportunity cost 85,000

xxxvii. Answer: A
Purchase price 15,000
Handling cost (20% x P15,000) 3,000
Total 18,000
Cost to make (21,200 – 8,000)* 13,200
Increase in unit cost if goods are purchased 4,800

*Fixed OH (12,000 x 2 ÷ 3) = 8,000

xxxviii. Answer: A
Cost to make:
Direct materials P10,000
Direct labor 5,000
Variable overhead 5,000
Avoidable fixed OH (20% x 30,000) 6,000
Relevant cost P26,000
Purchase costs (1,000 @ 30) 30,000
Decrease in profit in profit P 4,000

xxxix. Answer: B
Relevant costs to make per
unit: Direct materials 6.00
Direct labor 4.00
Variable overhead 1.00
Relevant cost – “to make” 11.00
Purchase price per unit 12.50
Increase in per unit cost if 1.50
purchased

xl. Answer: A
Direct materials 10,000
Direct labor 5,000
Variable overhead 5,000
Avoidable fixed overhead (30,000 x 0.2) 6,000
Total relevant cost 26,000
Purchase cost 30,000
Additional cost if purchased 4,000

xli. Answer: C
Direct materials 3.00
Direct labor 15.00
Variable overhead 6.00
Avoidable fixed cost 3.00
Total per unit 27.00
Number of unit x10,000
Total 270,000
Add savings from the manufacture of other product 45,000
Total relevant cost – make 315,000
Total purchase cost (10,000 x 30) 300,000
Advantage “Buy” 15,000

xlii. Answer: D
Though the problem deals with transfer of goods from one division to another division, the solution focuses on make on buy decision approach.
Purchase price, outside supplier 1.25
Variable cost to make (10,000 ÷ 10,000) 1.00
Additional unit cost to the company 0.25
Units to be purchased 10,000
Decrease in Dana‟s profit if goods are purchased 2,500

xliii. Answer: C
Total purchase cost (5,000 x 38,750
7.75) Less Relevant cost to
make
Direct materials @ 2.5 12,500
Direct labor @ 3.5 17,500
Variable overhead @ 1.5 7,500
Avoidable fixed cost @ 0.5 2,500
Opportunity cost 6,000 46,000
Net saving – purchase (7,250)
xliv. Answer: D
The solution is made in equation form, using y = a + bx for 2
alternatives: Let x = indifference point in units
Make: y = 150,000 + 11x
Buy: y = 60,000 +
12.875x
150,000 + 11x = 60,000 + 12.875x
1.875x = 60,000
x = 48,000

xlv. Answer: C
Direct materials 80,000
Direct labor 13,000
Variable overhead 40,000
Avoidable fixed overhead 20,000
Total relevant cost 153,000

xlvi. Answer: B
Soda Lemomade
Selling price 6.00 5.00
Variable cost 1.50 1.00
Contribution margin 4.50 4.00
Processing hours 3 4
CM/Hr 1.50 1.00

For the Lemonade to be as profitable as Soda, its contribution margin per hour should be P1.50.
Therefore the required selling price for Lemonade is P7, calculated as:
Contribution margin per unit (4 hours x P1.50) P6.00
Variable cost per unit 1.00
Selling price P7.00

xlvii.Answer: C
Product B has a greater contribution margin per unit (P15 - P12 = P3) than Product A (P12 - P10 = P2). The company should produce the maximum units it can sell of Product B (250,000) and
use the rest of the machine hour capacity to produce 100,000 units of Product A.

xlviii. Answer: D
Production order: Y, X
Product X: 60 ÷ 6 = 10
Product Y: 42 ÷ 3 = 14
Total capacity – MH 42,000
Machine hours devoted to Product Y (8,000 x 3) 24,000
Hours available to X 18,000
Production of X: 18,000 ÷ 6 =

3,000 xlix. Answer: B


Production order:
Blender Electric Mixer
Purchase price 20 38
Variable cost to make:
Direct materials 6 11
Direct materials 4 9
Overhead *(16 – 10) @ 6 12
Total ( 16) (32)
Additional cost if purchased 4 6
Additional cost per hour (Blender, 1 hr;
Mixer 2 hours) 4 3

Since it will cost Mary P4 per hour to buy Blender and only P3 if Electric Mixer is purchased, it will produce all of Blender‟s requirement and just purchase units of electric mixer that
cannot be accommodated by the remaining capacity.

Product:
Blender 20,000
Electric Mixer [50,000 – (20,000 @ 1)] ÷ 2 15,000
Purchase:
Electric Mixer (28,000 – 15,000) 13,000
l. Answer: A
CM – Product A 36/2 x 1,000 18,000
CM – Product B 45/3 x 1,000 15,000
Difference in contribution margin 3,000

li. Answer: A

Based on DLH
Products UCM DLH/unit CM/DLH Priority
A 9 1.0 9.0 1ST
B 10 1.5 6.67 2nd
C 8 2.0 4.00 3rd

Based on MH
Products UCM MH/unit CM/MH Priority
A 9 4.5 2.0 3rd
B 10 2 5.0 1st
C 8 2.5 3.2 2nd

lii. Answer: D
Plastic Metal
RC – make 11.00 13.00
RC – Buy 15.50 17.50
Additional Cost-Buy 4.50 4.50
Hours required/unit ÷3 ÷ 4.5
Additional cost /hr. 1.50 1.0
Priority 1st 2nd
Capacity (machine hours) 48,000
MH used - Plastic (7,000 x 3) 21,000
Available MH to Metal 27,000
MH used - Metal (6,000 x 4.50) (27,000)
Purchase of Metal (11,000 – 6,000) 5,000

liii. Answer: A
Home Deluxe Pro
Selling price 58 65 80
Direct materials (16) (20) (19)
Direct labor (10) (15) (20)
Variable overhead ( 8) (12) (16)
CM/unit 24 18 25
Processing hour(s) ÷1 ÷ 1.5 ÷2
CM/DLH 24 12 12.50
Profitability rank 1st 3rd 2nd

liv. Answer: B
Unit contribution margin:
Product A P12 – P10 P2
Product B P15 – P12 P3

Contribution margin per hour:


Product A P2 ÷ 0.8 P2.50
Product B P3 ÷ 1.25 P2.40

Total capacity in hours 350,000


Less hours used by Product A 200,000 x 0.8 (160,000)
Available hours for production of Product B 190,000
Less hours by Product B 152,000 x 1.25 (190.000)
Number of units to be
produced: 200,000
Product A
Product B 152,000

Product A has higher contribution margin per hour. The company should produce the maximum units it can sell of Product A and use the rest of the machine hour capacity to produce units of
Product A in order to maximize its profit.

lv. Answer: B
CM per hour:
Product X: 50/5 10
Product Y: 64/8 8
The 20,000 hours (0.8 x 25,000) will be devoted to the production
of X.
Total contribution margin: (20,000 x 10) + (5,000 x 8) 240,000

lvi. Answer: A
Selling price per unit – silver polish P40
Less variable costs:
Grit 337 (P20 ÷ 4) P5
Ingredients, direct labor and variable OH 25
Variable selling costs 3 33
Contribution margin per unit P
7

Minimum number of jars of silver polish to be produced:


Avoidable fixed costs ÷ Contribution margin per jar P56,000 ÷ P7 8,000

The solution used the selling price of P20 as cost of Grit337 because there was unlimited demand for the cleaning powder. If, however, the demand for the cleaning powder is limited, the
recommended solution would use P16 as the cost of Grit 337.

lvii. Answer: D
Increase in selling price 116 – 90 26
Additional processing cost 18
Addition profit per unit 8

lviii. Answer: C
Selling price after further processing P8.75
Selling price if not processed further 7.20
Additional sales per unit 1.55
Number of units 25,000
Additional total sales P38,750
Less additional processing costs 33,750
Increase in profit if the product is processed P 5,000
Because further processing will provide more profit per unit, the company should process
further.

lix. Answer: D
Additional sales (350,000 – 250,000) P100,000
Additional costs (75,000 + 15,000) 90,000
Additional profit P 10,000

lx. Answer: A
X Y
Additional sales value 10 34
Additional processing costs 15 30
Incremental (decremental) profit per unit (5) 4
If Product Y is processed further, profit will increase by P16,000 (4,000 x

4). lxi. Answer: A


Additional Sales Price (65 – 50) 15.00
Additional Cost (30 x 40%) 12.00
Additional profit 3.00

lxii. Answer: C
Product to be processed further: Prod M Prod N
Final selling price 31 23
Selling price at split-off point 25 19
Increase in selling price 6 4
Units 15,000 30,000
Total increase in sales 90,000 120,000
Additional processing costs 100,000 110,000
Increase (decrease) in profit (10,000) 10,000

lxiii. Answer: C
Revenues P104,000
Avoidable costs:
Cost of goods sold P 64,000
Avoidable expenses (P6,000 + P8,000) 14,000 78,000
Segment margin P
26,000
A segment is a potential candidate for elimination if its revenues are less than its avoidable costs. This is not the case for this segment. The company will lose P26,000 of income if this segment is
eliminated.

lxiv. Answer: A
Avoidable fixed cost (benefit) 210,000
Lost contribution margin 240,000
Decrease in profit 30,000

lxv. Answer: D
The question did not require any computation. If Mina Co. drops the Gold Ore, it will lose the segment margin of P1,200,000, a decrease in Mina Co.‟s income. The amount of direct fixed expenses
that would be eliminated were previously deducted from contribution margin, and therefore, not considered in the determination of the effect on income.

lxvi. Answer: B
Avoidable common expenses (45,000 – 20,000) P 25,000
Segment margin lost 32,000
Decrease in profit P (7,000)

lxvii.Answer: A
Avoidable fixed expenses:
Manufacturing (150,000 – 105,000) 2 90,000
Selling (30,000 x 0.10 x 2) 6,000
Start up cost (additional fixed expense (
8,000)
Net avoidable costs 88,000
Indifference point 88,000 ÷ (22-14) 11,000 units
At 11,000 unit level (2 months), the contribution margin equals the avoidable
costs.

lxviii. Answer: D
Total Savings 5 year (125,000 – 100,000 ) 125,000
5 Less:
Additional depreciation (90,000 – 50,000) (40,000)
Loss on sale of old machine (5,000 – 50,000) (45,000)
Increase in profit 40,000

lxix. Answer: A
Lease arrangement:
Rental income (5 years) 48,000
Cost of repairs, insurance and property taxes 10,000
Net income 38,750

Sale arrangement:
Net proceeds (25,000 x 0.95) 23,750
Differential income –lease 15,000

lxx. Answer: A
Additional contribution (60,000 x 0.25 x 14) 210,000
Additional fixed selling costs 80,000
Additional profit 130,000

Selling price 32.00


Variable expenses:
Materials 10.00
Direct labor 4.50
Variable overhead 2.30
Variable selling costs 1.20 18.00
Unit contribution margin 14.00
lxxi. Answer: C
Direct materials 10.00
Direct labor 4.50
Variable OH 2.30
Variable selling cost 3.20
Import duties 1.70
Permits and licenses (9,000 ÷ 20,000) 0.45
Minimum selling price 22.15

Import duties are assumed to be paid by Adrenal Company because of the nature of the sale.

lxxii.Answer: D
The relevant cost in selling the units on hand (inferior quality) is P1.20, the variable selling costs the production costs, though variable, are considered irrelevant because they are historical (sunk)
costs.

lxxiii. Answer: C
Avoidable fixed costs:
Manufacturing (0.40 x 50,000) 20,000
Selling (35,000 x 0.20) 7,000
Total 27,000

Contribution margin if the company has to operate (60,000 ÷ 6 x 0.30 x 42,000


14) Disadvantage, closing the plant (15,000)

lxxiv. Answer: A
Direct materials 10.00
Direct labor 4.50
Variable overhead 2.30
Avoidable fixed overhead (0.75 x 5) 3.75
Avoidable variable expense (1.20 x 1 ÷ 3) 0.40
Relevant cost – Make 20.95

lxxv. Answer: A
Batch (each 50 units) Cum Ave. Hrs
1 14.25
2 11.40
4 9.12
8 7.296
16 5.8368

Total Hours required: 16 x 5.8368 = 93.4


lxxvi. Answer: D
Materials (800 x 40.50) 32,400
Direct labor (93.40 x 120) 11,208
Variable OH (93.40 x 100) 9,340
Total 52,948

lxxvii. Answer: A
Production cost – 750 units:
Materials (750 x 40.00) 30,375
Direct labor (93.40 – 14.25) 120 9,498
VOH (93.40 – 14.25) 100 7,915
Total 47,780
Purchase cost (750 x 75)
Advantage – make 56,250
8,470

lxxviii. Answer: A
Sales price to Kay Corp. 68,400
Rework costs:
Direct materials 6,200
Direct labor 4,200
Variable OH (4200 x 50%) 2,100 (12,500)
Commission (68,400 x 0.03) ( 2,052)
Before – tax peso contribution 53.848

lxxix. Answer: A
Regular price 62,500
Deduct:
2% commission (62,500 x 0.02) 1,250
Sales discount (62,500 x 0.02) 1,250 2,500
Net price 60,000
Less additional conversion
costs: Direct materials 2,850
Direct labor 3,300
Variable OH - 50% 1,650 7,800
Net before – tax contribution 52,200

lxxx. Answer: D
Cost of rework 6,200
Direct labor 4,200
Variable OH (4,200 x 0.50) 2,100
Total 12,500
Commission [0.03 (12,500 0.97)] 387
Total 12,887

lxxxi. Answer: A
Sales price 52,000
Less: Commission (52,200 x 0.03) 1,560
Net contribution 50,440

lxxxii. Answer: C
Department 3 has constraint in labor of 750.
hours Dept. 2 Dept. 3 Dept. 4
Dept. 1
Available DLH 3,700 4,500 2,750 2,600
DLH required
401 1,000 1,500 1,500 500
402 400 800 -- 800
403 2,000 2,000 2,000 1,000
Total 3,400 4,300 3,500 2,300
Excess 300 200 ( 750) 300
(Constraint)

lxxxiii. Answer: A
The available machine hours are sufficient to produce the estimated monthly sales. The schedule for monthly production should consider maximizing the use of available direct labor hours in
Department 3 because it is the only one with constraint.
Dept. 1 Dept. 2 Dept. 3 Dept. 4
Available MH 3,000 3,100 2,700 3,300
MH required
401 500 500 1,000 1,000
402 400 400 -- 800
403 2,000 2,000 1,000 1,000
Total 2,900 2,900 2,000 2,800
Excess 100 200 700 500
(Constraint)

Total machine hours required by monthly unit sales: (2,900 + 2,900 + 2,000 + 2,800) 10,600
lxxxiv. Answer: B
The table showing the comparison of available hours and required hours to produce all the required units in number 82 indicated that Department 3 is short by 750 hours. Any excess direct labor
hours in the other departments cannot be switched to Department 3.

lxxxv. Answer: B
The production plan that will maximize monthly profit should be based on the profitability of the three products in terms of the use of direct labor hours in Department 3.
PRO DUCT S
401 403 405
Selling price per unit P196 P123 P167
Variable unit costs
Direct material 7 13 17
Direct labor 66 38 51
Variable overhead 27 20 25
Selling expenses 3 2 4
Total variable cost 103 73 97
Unit contribution margin P 93 P 50 P 70
No. of DLH required – 3 - 2
Dept
3
Contribution margin per P 31 - P 35
DLH
Based on the above schedule, Product 405 is more profitable per hour than Product 401‟s and, therefore, all of the units required for Product 405 should be produced. Product 403 would not use
any direct labor hours in Department 3 and so all of the required units for Product 403 can be produced.

Available direct labor hours – Department 2,750


3
Hours used by Product 405 1,000 x 2,000
2
Available hours for Product 401 750

Production units – Product 401 250 x 3 750

Production:
Product 401 250
Product 403 400
Product 405 1,000

Alternative Solution:
Since Product 401 is the less profitable per DLH, Product 403 and 405 will be produced in full and Product 401 will be partially produced.

Total required units, Product 401 500


Equivalent units based on constraint 750 ÷ 3 250
Production of Product 401 250

Alternative question: What is the maximum monthly contribution margin that Constraint Company can earn?
Product 401 250 @ P93 P 23,250
Product 403 400 @ P50 20,000
Product 405 1,000 @ P70 70,000
Total contribution margin P113,250

lxxxvi. Answer: B
Costs incurred to make the order:
Material (5,000 x 40) P200,000
Labor (5,000 x 72) 360,000
Incremental fixed cost (special device) 40,000
Costs to be incurred P600,000

Decrease in costs for standard products:


Material (0.5 x 160,000) P 80,000
Labor (0.5 x P180,000) 90,000
Other (0.5 x P18,000 9,000
Decrease in costs P179,000
Net incremental costs P421,000

The amounts for depreciation, rent, and heat and light are assumed to be not affected by the special order. There is no information provided as to how power cost was exactly

incurred. lxxxvii. Answer: D


Costs to be incurred for special order P600,000
Fixed costs:
Depreciation (0.5 x 72,000) P36,000
Power (0.5 x 8,000) 4,000
Rent (0.5 x 20,000) 10,000
Heat and Light (0.5 x 2,000) 1,000 51,000
Total cost P651,000
The amount of fixed costs allocated to special order would be the costs that should have been assigned to the standard sales that would be cancelled.

lxxxviii. Answer: B
Decrease in sales of standard products0.50 x P250,000
500,000 Less variable costs:
Material (160,000 x 0.5) P80,000
Labor (180,000 x 0.5) 90,000
Other (18,000 x 0.5) 9,000 179,000
Opportunity costs P 71,000

lxxxix. Answer: D
Special sales (5,000 x 140) P700,000
Variable costs 600,000
Contribution margin from special sale 100,000
Less opportunity costs 71,000
Increase in profit P 29,000

xc. Answer: C
Total overhead rate per box P150
Less fixed overhead allocated per boxP10,000,000 ÷ 100,000 boxes 100
Variable overhead rate per box P 50

xci. Answer: C
The cost of materials saved by a decision of purchasing the tubes: is P300 x 0.20 = P 60

xcii. Answer: B
The relevant cost to make the tubes by Verbatim should equal the amount of cost savings as
follows: Savings on materials 0.2 x P300 P 60
Labor 0.1 x P200 20
Overhead 0.1 x P 50 5
Total savings (relevant cost) P 85
The maximum amount that Verbatim is willing to pay per box of 24 tubes must be P85.

xciii.Answer: B
Cost of making 125,000
boxes: Variable costs 10,625,000
125,000 x 85
Additional fixed costs 1,000,000
Total 11,625,000

xciv. Answer: C
Total purchase cost 125,000 x 900 11,250,000
Total cost to make 125,000 x 85 11,625,000
Savings if purchased 375,000

xcv. Answer: A
Fixed costs:
Manufacturing 3,000 x 1,200 P3,600,000
Marketing 3,000 x 1,400 4,200,000
Total P7,800,000

Selling Price P 7,400


Less Variable costs:
Direct materials P1,000
Direct labor 1,500
Variable overhead 500
Marketing costs 500
Total 3,500
Unit contribution margin P 3,900

Breakeven units 7,800,000 ÷ 3,900 2,000

units xcvi. Answer: C


In as much that there would be no change in the amount of fixed costs, the recommended solution was made by just comparing the amounts of contribution margin based on the revised
data and the original information:

Contribution margin based on new estimates 3,500 x (6,500 – 3,500) 10,500,000


Contribution margin based on current estimates
Decrease 3,000 x (7,400 – 3,500) 11,700,000
Decrease in profit ( 1,200,000)

Alternative Solution:
Total contribution margin 3,000 x (7,400 – 3,500) 11,700,000
Less Fixed costs 7,800,000
Current profit 3,900,000

Total contribution margin at reduced price 3,500 x (6,500 – 10,500,000


3,500)
Less Fixed costs 7,800,000
Revised profit 2,700,000
Current profit 3,900,000
Decrease in profit ( 1,200,000)
xcvii. Answer: B
Fixed fee P 500,000
Fixed overhead reimbursement 500 x 1,200 600,000
Total 1,100,000
Less lost contribution margin on regular customers (500 x 3,900) 1.950,000
Decrease in profit P( 850,000)
The reimbursement for fixed overhead is an income for Medical Hospital Company because the special order does not entail additional fixed

overhead. xcviii. Answer: C


Direct materials 1,000
Direct labor 1,500
Variable overhead 500
Shipping cost 750
Cost of obtaining the order 40,000 ÷ 1,000 40
Minimum selling price 3,790

xcix. Answer: D
All the production costs, both variable and fixed, are no longer relevant because they are sunk costs. To be relevant to a decision, the cost must be both valid and relevant. Therefore, the only
relevant cost is the variable marketing cost, because if the units will be sold through regular channel, P500 will be incurred.

c. Answer: D
The maximum price at which the price charged by the contractor would indifferent to the cost to make the hoist is the total differential cost or avoidable cost.
Direct materials 1,000
Direct labor 1,500
Variable overhead 500
Avoidable fixed overhead 1,200 x 0.30 360
Avoidable variable marketing cost 500 x 100
0.2
Maximum purchase price 3,460

ci. Answer: A
Direct materials 1,000
Direct labor 1,500
Variable overhead 500
Avoidable marketing costs 100
Opportunity cost [800 x (9,000 – 5,500 – 1,000)] ÷ 1,000 2,000
Maximum purchase price 5,100

A better understanding of the solution can be made by drawing a schedule to compute income for this alternative and compare it with the income shown in solution for Question No. 97
as follows:
Modified Regular
Sales 7,200,000 22,200,000
Variable production costs:
In house production
(2,000 x 3,000) 6,000,000
(800 x 5,500) 4,400,000
Contractor‟s cost 1,000 x 5,100 5,100,000
Variable marketing costs
Regular (2,000 x 500) + (1,000 x 400) 1,400,000
Modified (800 x 1,000) 800,000
Fixed costs . . 7,800,000
Profit 2,000,000 1,900,000

Total profit (2,000,000 + 1,900,000) 3,900,000

cii. Answer: A
Direct material (6 lbs.  P1.50) P9.00
Direct labor (0.25 hr.  P7) 1.75
Direct machine cost (P10/blanket) 10.00
Variable overhead (0.25 hr.  P3) 0.75
Administrative costs (P2,500/1,000) 2.50
Minimum bid price P24.00

ciii. Answer: B
Using the full-cost criteria and the maximum allowable return specified, Marcus Fibers‟ bid price per blanket
would be: Relevant costs (from Requirement 1) P24.00
Fixed overhead (0.25 hr.  P8) 2.00
Subtotal P26.00
Allowable return (0.15*  P26) 3.90
Bid price P29.90

*0.09/(1 – 0.40) = 0.15

You might also like